Corporate Finance-munotes

Page 1

1Unit-1
NATURE AND SCOPE OF FI NANCIAL
MANAGEMENT
Unit Structure:
1.0 Objectives
1.1 Introduction
1.2 Meaning and Definition of Financial Management
1.3 Scope of Financial Management
1.4 Importance of Financial Management in Business
1.5 Qualities o f a Successful Finance Manager
1.6 Functions of Financial Controller
1.7 Goals / Objectives of Finance Management
1.8 Exercises
1.0 LEARNING OBJECTIVES
The present chapter attempts to:
Provide familiarisation with financial objectives and goals of a
firm.
Develop conceptual framework of financial management.
Focus on nature, and scope of financial management.
Explaining the role of finance function.
Discuss the role of finance manager.
1.1 INTRODUCTION
Finance touches every aspect of our life and hol ds the key to
all activities. It has been described as the life blood of any
business. The blood in our body needs to be regulated to ensure
smooth circulation for healthy survival. Management of finance in
anoptimal manner is inevitable for success of an y business. The
finance function has been defined differently by different writers and
differently over time. According to G.L. Jones, the simplest way of
understanding finance is to say that finance is what finance does.
L.J. Gitman has defined finance as the art and science of managing
money. The only conclusion one may make with respect to finance
is that it has a marvellous ability to evoke different concepts in the
minds of men.munotes.in

Page 2

21.2 MEANING AND DEFINITION OF FINANCIAL
MANAGEMENT
Financial management means money management.
Financial management is concerned with the planning and
controlling of the financial resources of the business firm. The term
financial management has emerged from the generic discipline of
management. As an academic discipline, the subject of financial
management has undergone radical changes in relation to its
scope, functions and objectives. In the past, the financial
management was confined to rising of the funds and its procedural
aspects. In the broader sense, it is now concern ed with the
optimum use of financial resources in addition to its procurement.
Therefore, financial management is that part of management which
is concerned mainly with:
1. Fund Raising: raising the right type of funds in the most
economic and suitable ma nner.
2. Use of Funds: using the funds in the most profitable and safest
possible manner.
According to James Van Horne,
“Financial management connotes responsibility for obtaining
and effectively utilizing funds necessary for the efficient operation of
an enterprise.”
According to I.M. Pandey:
“Financial management is that managerial activity which is
concerned with the planning and controlling of the firm’s financial
resources”.
Financial management provides the best guide for future
resource allocatio n by the firm. It performs facilitation, reconciliation
and control function in an organisation. It permits and recommends
investment where the opportunity is greatest. Financial
management produces relatively uniform yardsticks for judging
most of the ent erprise’s operations and projects. It is continually
concerned with an adequate rate of return on investment which is
necessary to assure the successful survival of an enterprise. The
problem of attracting new capital and providing funds for capital
needs is solved if the return on investments is adequate. Because it
Iscontinuing drawing attention to such matters, financial
management is essential to effective top management.
Definitions of Financial Management
The simple definition of Financial Manageme nt is `the ways
and means of managing money’. This statement can be furthermunotes.in

Page 3

3expanded to define Financial Management: the determination,
acquisition, allocation and utilization of the financial resources with
the aim of achieving the goals and objectives of the enterprise.
According to Archer and Ambrosia:
“Financial management is the application of the planning and
Control functions to the finance function”.
Joseph and Massie:
“Financial management is the operational activity of a
business that is respo nsible for obtaining and effectively utilizing
the funds necessary for efficient operation”.
Raymond Chambers:
“Financial management may be considered to be the
management of the finance function. It may be described as
making decisions on financial matte rs and facilitating and reviewing
their execution. It may be used to designate the field of study which
lie beneath these processes”.
1.3 SCOPE OF FINANCIAL MANAGEMENT
All decisions that have monetary benefit scome under the
purview of financial manageme nt. There are basically, two
approaches for understanding the scope of financial management:
one is traditional approach and the second one is the modern
approach.
1. Traditional approach: Traditional approach views the scope of
finance function in a narr ow sense of arrangement of funds by
business firm to meet their financing needs. Hence, the following
three inter -related aspects of raising and administering financial
resources were covered:
a)Arrangement of finance from institution;
b)Raising funds in the capital market through financial instruments
including the procedural aspects;
c)Legal and accounting aspects involved for raising finance for the
firm.
The traditional approach was criticized for the reasons:
a)It emphasis only the issues rela ting to procurement of funds and
ignored the issues related to internal financial decisions.
b)It focused only on the problems related to corporate entities
ignoring the non -corporate bodies. The scope of financialmunotes.in

Page 4

4management was confined only to a parti cular segment of
business enterprises.
c)It laid more emphasis on the onetime events (episode) such as
promotion, incorporation, reorganization, etc., taking place in the
corporate life of the concern/ignoring the day -to-day financial
problems of the con cern.
d)The focus was more on long term financing. Working capital
management was considered to be outside the purview of
finance function.
According to Solomon, the traditional approach has ignored
the central issues of financial management which compr ise the
following:
i)Should the enterprise commit capital funds to certain purpose?
ii)Do the expected returns meet financial standards of
performance?
iii)How should these standards be set and what is the cost of
capital funds to the enterprise?
iv)How does the cost vary with the mixture of financing methods
used?
Therefore, the traditional approach while ignoring the above
crucial aspects implied a very narrow scope for financial
management. These defects were taken care by the Modern
approach.
2.Modern Approach: The traditional approach focused on
sources of funds and was too often largely concerned with specific
procedural details. Experts pointed out the following two defects of
traditional approach:
i)It does not recognize the relationship b etween financing mix and
the cost of capital and fails to solve the problems relating to
optimum combination of finance, and
ii)It also fails to deal with the problems relating to the valuation of
the firm and the cost of capital.
The modern approach ai ms at formulating rational policies
forthe optimal use, procurement and allocation of funds; unlike the
traditional approach which has focused only on the sources of
funds and their procedural details. The modern approach apart for
covering the acquisitio n of external funds; includes the efficient and
wise allocation of funds for various uses. Emphasis has shifted
from a detailed analysis, of operating procedures in the acquisition,
custody and disbursement of funds to the formulation of rationalmunotes.in

Page 5

5decisions on the optimal use and allocation of funds. Financial
decision making has become fully integrated in more advanced
companies with top management policy formulation via capital
budgeting, loan range planning, evaluation of alternate uses of
funds, and esta blishment of measurable standards of performance
in financial terms.
In the words of Solomon, a financial manager should know
thefollowing:
i)How large should an enterprise be and how fast should it grow?
ii)In what form should it hold its assets?
iii)What should be the composition of its liabilities?
Thus, the modern approach views the term financial
management in a broad sense and provides a conceptual and
analytical framework for financial decision making. Therefore,
financial management, in the modern sense of the term, can be
related to three major decision making areas. They are as follows:
1.Investment Decision i.e. Where to invest funds and in what
amounts?
2.Financing Decisions i. e .Where to raise funds from and in what
amount?
3.Dividend Decisions i. e how much of profits should be paid by
way of dividends and how much should be retained in the
business?
All the above three decisions contribute towards the goal of
wealth maximization.
1. Investment Decisions: Investment decisions involve identifying
the asset or projects in which the firms limited resources should be
invested. It involves the major task of measuring the prospective
profitability of investment in assets of the company or in new
projects. The decisions relating to ac quisition of fixed assets
investment are known as capital budgeting decisions and the
decisions relating to current assets investment are known as
working capital management decisions. Capital budgeting decisions
relate to selection of an asset or investme ntproposal or course of
action which have hot long term implications on the cash flows and
profitability of such investment. It helps in judging whether it is
financial feasible to commit funds in future. An important aspect of
working capital, the profit ability would be adversely affected,
whereas with too inadequate working capital, it would be unable to
meet its financial commitment on time and thereby invite the risk of
insolvency. The investment in the fixed assets of the companymunotes.in

Page 6

6determines the produc tion capacity of the company. The production
should be sufficient to demand in the market. Production should not
fall short or be too excessive in relation to the demand for the
product in the market. Further, the fixed assets must be productive
enough to ensure the returns expected from such investment. This
should be supported by sufficient investment in the working capital
assets. The working capital assets should be adequate enough to
maintain sufficient liquidity to augment the sales level. Investment
decisions yield returns in future. Future performance is subject to
uncertainty and risk. Therefore, investment decisions require
careful analysis before substantial amounts are committed in fixed
assets. The investment decisions having long term implicati onsand
affects the cash inflows in the years to come. Hence any wrong
decision taken in the initial year, would adversely affect the future
profitability and growth. Hence appropriate techniques need to be
adopted for proper evaluation of investment decis ions.
2. Financing Decisions: Financing decisions involve deciding on
the most cost effective method of financing the chosen
investments. Financing decisions relate to the financing pattern of
the firm. It involves in deciding as to when, where and how to
acquire the funds to meet the firm’s investment needs. Different
sources of finance have different advantages with different degree
of risks. Hence it becomes imperative to decide as to how much
finance is to be raised and from which sources. The prime ob jective
of financing decisions is to keep the cost of finance at the minimum
with maximum utilization of funds. Primarily, there are two main
sources of finance: one is the owned funds and second is the
borrowed funds. Owned funds are the shareholder’s mon ies on
which dividend are paid. Dividend payment depends upon the
profitability of the company and is not binding. There is no
commitment involved in the shareholders funds. On the other hand,
borrowed funds involve fixed commitments; their repayments are
secured by a charge created on the assets and interest payments
are obligatory irrespective of the profits or losses of the company.
Hence, it increases the financial risk of the company. The borrowed
funds are relatively cheaper, but entail a certain degr ee of risk,
therefore, due prudence must be exercised while determining the
mix of owned and borrowed funds.
3. Dividend Decision: Dividend Decisions involve the decisions as
regards what amount of profits earned should be distributed by way
of dividends and what amount should be retained in the business.
Dividend policy is to be decided having regard to it’s implicate on
the shareholder wealth in the firm. The aim is to decide an optimum
dividend policy which would maximize the market price of shares.
This is a crucial decision as it determines the reputation of the
management of the company and therefore, the market value of the
shares. If the management decides to retain profits, it should bemunotes.in

Page 7

7able to generate adequate returns (by investing such retained
profits), which should be much more that what the, shareholders
could have got, had they received the dividends and invested the
amount elsewhere. If the management is not able to generate
adequate returns on reinvestment of retained profits, then it shou ld
prefer to pay dividends rather than retaining the profits. Therefore,
the two important factors which affect the dividend decisions are:
firstly, the investment opportunities available to the firms and
secondly, the opportunity rate of return of the shareholders. The
topic has been dealt in more details in the subsequent chapters of
this book.
1.4 IMPORTANCE OF FINANCIAL MANAGEMENT IN
BUSINESS
The importance of financial management is known from the
following aspects:
1. Applicability –The principles of finance is applicable wherever
there is cash flow. The concept of cash flow is one of the central
elements of financial analysis, planning, control and resource
allocation decisions. Cash flow is important because financial
health of the firm depends o ni t sa b i l i t yt og e n e r a t e sufficient
amounts of cash to pay its employees, suppliers, creditors and
owners. Any organization, whether motivated with earning of profit
or not, having cash flow requires to be viewed from the angle of
financial discipline. Th erefore, financial management is equally
applicable to all forms of business like sole traders, partnerships,
companies. It is also applicable to non profit organizations like
trusts, societies, governmental organizations, public sector
enterprises etc.
2. Chances of Failure –A firm having latest technology,
sophisticated machinery; high calibre marketing and technical
experts etc. may fail to succeed unless its finances are managed
on sound principles of financial management. The strength of
business lik es in its financial discipline. Therefore, finance function
is treated as primary, which enable the other functions like
production, marketing, purchase, personnel etc. to be more
effective in achievement of organizational goals and objectives.
3. Return on investment –Anybody invests his money will mean to
earn a reasonable return on his investment. The owners of
business try to maximize their wealth. It depends on the amount of
cash flows expected to be generated for the benefit of owners, the
timing of these cash flows and the risk attached to these cash
flows. The greater the time and risk associated with the expected
cash flow, the greater is the rate of return required by the owners.munotes.in

Page 8

8The Financial management study the risk-return perception of the
owners and the time value of money.
1.5 QUALITIES OF A SUCCESSFUL FINANCE
MANAGER
The job of a finance manager is full of duties and
responsibilities. He has to perform various duties connected with
finance. In order to perform the finance duties successfu lly, a
finance manager should be competent. He should possess the
following qualities:
1. Personality is the sum total of physical and mental qualities. A
finance manager should have a pleasing personality. Good
height, good physique, good appearance woul d be an asset to a
finance manager. He should be physically and mentally healthy
enough to bear the strain of finance in an organization.
2. The job of a finance manager involves analytical work. He should
have a high degree of intelligence to understand thefinance
problems immediately. An intelligent finance manager can
control the finance properly.
3. A finance manager should take initiative in performance of work.
He should do the job at his own i.e. without being told by others.
4. A finance manage r should have vast fund of power of
imagination to his credit. He should have a research mind which
is very creative. He should be able to bring innovation in
financial management of an organization.
5. A finance manager should have self confidence to fac et h e
challenges involved in his job.
6. A finance manager is a leader of financial administration. He
should have an effective Communication Skill. He should
understand the problems of his subordinates and communicate
instructions to solve them.
7. The job of a finance manager involves decision making. He has
to take various decisions which have financial implications on
theworking of the organization. He should have the quality to
judge the situation and take right decision accordingly.
8. He should b e honest in his job. Finance requires utmost honesty
on the part of the manager and the subordinates also.munotes.in

Page 9

99. He should have an administrative skill to administer the finance
function. He should be able to plan, organize, direct, control and
coordinate th e activities of the finance area. He has to see that
the financial decisions are properly implemented.
10. A finance manager should be self -disciplined. He should be
able to enforce discipline in the organization.
11. A finance manager should have patien ce. He should not take
hasty decisions which have adverse impact on the financial
health of the organization. He should listen to the views of
others.
1.6 FUNCTIONS OF FINANCIAL CONTROLLER
The important functions of a financial controller in a large
business firm consist of the following:
1. Provision of Capital –To establish and execute programmes for
the provision of capital required by the business.
2. Investor Relations –To establish and maintain an adequate
market for the company’s securities and to maintain adequate
liaison with investment bankers, financial analysis and
shareholders.
3. Short -term Financing –To maintain adequate sources for
company’s current borrowing from commercial banks and other
lending institutions.
4. Banking and Custod y–To maintain banking arrangement, to
receive, have custody of and disburse the company’s monies and
securities.
5. Credit and Collections –To direct the granting of credit and the
collection of accounts due to the company, including the
supervision of required special arrangements for financing sales,
such as time payment and leasing plans.
6. Insurance –To provide insurance coverage as required.
7. Investments –To achieve the company’s funds required and to
establish policies for investment in pen sion and other similar trusts.
8. Planning for Control –To establish, coordinate and administer an
adequate plan for the control of operations.munotes.in

Page 10

109. Reporting and interpreting –To compare performance with
operating plans and standards, and to report and interpret the
results of operations to all levels of management and to the owners
of the business.
10. Evaluating and Consulting –To consult with all segments of
management responsible for policy or action concerning any phase
of the operation of the bus iness as it relates to the attainment of
objectives and the effectiveness of policies, organization structure
and procedures.
11. Tax Administration –To establish and administer tax policies
and procedures.
12. Government Reporting –To supervise or coo rdinate the
preparation of reports to government agencies.
13. Protection of Assets –To ensure protection of assets for the
business through internal control, internal auditing and proper
insurance coverage.
14. Economic Appraisal –To appraise continuo usly economic,
social forces and government influences, and to interpret their
effect upon the business.
15. Managing Funds –To maintain sufficient funds to meet the
financial obligations.
16. Measuring of Return –To determine required rate of return f or
investment proposals.
17. Cost control –To facilitate cost control and cost reduction by
establishment of budgets and standards.
18. Price Setting –To supply necessary information for setting of
prices of products and services of the concern.
19. F orecasting Profits –To collect relevant data to make forecast
of future profit levels.
20. Forecast Cash flow –To forecast the sources of cash and its
probable payments and to maintain necessary liquidity of concern.
1.7 GOALS / OBJECTIVES OF FINANCE
MANAGEMENT
Many of the well known authors on the subject have
highlighted the following two important goals of financial
management. They are as follows:munotes.in

Page 11

111. PROFIT MAXIMIZATION:
The objective of making profit is a commercial imperative.
Profit generation i s essential for survival and growth of the
business. Profit generation is also regarded as a measure of
success of the business. Profit is an important yardstick for
measuring the economic efficiency of any firm. Any business would
be making the use of eco nomic and human resources available to
generate profits. The cost of these resources is required to be met
from the revenue generated from the use of these resources and
the surplus remaining would be needed for the growth and
expansion of the company. It is only an efficiently run business
which can afford to meet the cost of resources and generate profits.
Therefore, the survival and growth of any business depends upon
its ability in earnings profits. It is therefore contended that profit
maximization is one of the primary goals of the organization without
which the survival of the organization itself is threatened.
THE DRAWBACKS OF THE GOAL OF PROFIT
MAXIMIZATION
Although profit is an important yardstick for measuring the
economic efficiency of any firm, yet it has got certain limitations
which are listed below:
1. It ignores the risk which is associated with the investment in such
profitable ventures. It ignores the risk or uncertainty ofexpected
returns or benefits. Risk is defined as the chance that theactual
outcome of a decision may differ from the expected outcome and in
finance; risk investment is one whose potential returns are
expected to have a high degree of variation or volatility. Some
investment swith high profits potential ,having a high degree of risk
associated with it .W h e np r o f i tm a x i m i z a t i o ni sa i m e da st h em a i n
objective, all profitable investment projects are accepted without
having regard to the risk factor. An investment may have profit
potential but may not be worth the risk.
2.The objective of profit -maximization assumes the existence of
perfect market conditions in which various resources are efficiently
managed. However, modern markets suffer from many
imperfections. It leads to inequitable distribution of income and
wealth.
3. It ignores the time value of money without having any regard to
the timings of costs and returns. It takes into account only the size
of the profits without considering the timings of the prospective
earnings.
4. Profit maximization as an objective is considered to be vague
and ambiguous. It does not define adequately as to what profits are
what profits to be considered, whether from the point of view ofmunotes.in

Page 12

12funds employed or from the shareholders point of view, or short
term or long term profits etc.
5.Profit maximization as an objective ignores other important
aspects of financing e.g. borrowing capacity etc.
6. The objective of profit maximization focuses on interests of the
owners alone and ignores the interest of other interested parties
such as emp loyees, consumers, government and society in
general.
7. The perception of the management as regards profit
maximization substantially differs from the perception of the
shareholders.
Another variant of profit maximization is to consider the rate
of retu rn on investment. If the rate of return on investment is higher
than the cost of funds, then such investment opportunities can be
undertaken.
2. Wealth Maximisation:
According to this objective, the owners of the company i.e.
the shareholders are more int erested in maximizing their wealth
rather than in profit maximization. Maximization of the wealth of the
shareholders means maximizing the net worth of the company for
itsshareholders. This reflected in the market price of the shares
held by them. Therefo re, wealth maximization means creation of
maximum value for company’s shareholders which mean
maximizing the market price of the share. Wealth maximization
refers to the gradual increase in value of the net assets of the
organization. Profit generation add s to the increase in the value of
the net assets of the organization. With greater profits, the EPS
(earnings per share) goes up; resulting inan increase in the value
ofthe net assets belonging to the shareholders of the company.
The market price of th e shares is an important indicator of
thewealth maximization of the organization. Wealth maximization is
thenet present value of a financial decision. Net present value is
thedifference between the gross present value of the revenue
generated from such decision and the cost of such decision. A
financial action with a positive net present value creates wealth and
therefore is desirable. The total cash inflows over the years in terms
of present value must be greater the outflows of cash invested for
genera ting such cash inflows. This results in financial advantage
leading to increase in the value of net assets. The increase in the
value of net wealth should in turn help in generating greater volume
of profits. This action results in financial gains to the s hareholders
increasing the earnings per share.munotes.in

Page 13

13Prof. Solomon has suggested wealth maximization as the
best criterion. According to him “Wealth or net present worth is the
difference between gross present worth and the amount of capital
investment required to achieve the benefits. Any financial action
which creates wealth or which has a net present worth above zero
isa desirable one and should be undertaken. Any financial action
which does meet this test should be rejected”.
Solomon states that wealth max imization provides an
unambiguous measure of what financial management should seek
to maximize in making investment and financing decisions.
Future earnings of a company are subject to uncertainties
and exposed to risk. Financial decisions for which the
consequences are known at a later date may either result in
increasing or decreasing the net wealth of the shareholders.
Unforeseen economic and social conditions may adversely affect
the company. Hence the process of wealth generation is a difficult
task.
Therefore, the goal of wealth maximization implies a long
term perspective of the goal. The interest of the management in
maximizing the market price of the share is compatible with that of
the shareholders’ interest. This helps the management in allocat ing
the resources in the best possible manner balancing the risks and
the returns.
THE MERITS OF THE GOAL OF WEALTH MAXIMIZATION
ARE AS FOLLOWS:
1.It is a very effective and meaningful criterion to measure the
performance of the company.
2.The objecti ve of wealth maximization is consistent with the
objective of maximization of the shareholders’ economic
welfare.
3.The objective is also consistent with the objective of perpetual
survival of the company and its long term profitability.
4.It is operat ionally feasible and logical.
5.It includes the motive of profit maximization as it emphasis on
maximization of long term profitability and ensures maximum
return on owners’ investment.
6.The objectives allow for timings of profits and also consider the
timings of perspective benefits.
7.It ensures fait return on the investments, and takes into account
the uncertainty of the benefit also.munotes.in

Page 14

148.It offers rational guidelines for effective use of the resources
available.
THE DRAWBACKS OF THE GOAL OF WEAL TH
MAXIMIZATION
i)The basic assumption is that there an efficient capital market
wherein the market price of the share is truly reflected. This
assumption seldom holds in real practice.
ii)The market price is influenced by various economic and politica l
factors which are difficult to anticipate and judge.
iii)The various parties having their stake in the company have
conflicting interests and therefore difficult to reconcile their
divergent views.
OTHER GOALS OR OBJECTIVES OF FINANCIAL
MANAGEMENT:
i)To ensure adequate returns to the shareholders which should
be fair in the given market conditions.
ii)To contribute to the operational efficiency of all other areas of
management.
iii)To infuse financial discipline in the organization.
iv)To build up a strong financial base so that the enterprise can fall
back upon its reverses during lean years and withstand the
shocks of the business.
1.8 EXERCISES
1. Define the scope of financial management. What role should
the financial manager plan in the m odern enterprise?
2. How should the finance function of an enterprise be
organized? What functions are performed by the financial
officers?
3. State the scope of Financial Management.
4. State and explain the main functions of a finance manager.
5. Explain the role of finance manager in a large corporate
enterprise.
6. What are the functions of Financial Management?
7. “The goal of profit maximization does not provide us with an
operationally useful criterion.” Comment on this statement.
8. What is o bjective of profit maximization pool? How is its
different from objective of profit maximization?munotes.in

Page 15

159. How does the modern finance manager differ from the
traditional finance manager?
10. Discuss the contents of modern finance functions.
11. Discuss the n ature and scope financial management.
12. Discuss the nature of financial management asa staff of line
functions ?
13. Describe the functions of finance. In what ways, are these
functions related to possible finance objectives of a company?
14. Explain the nature and scope of finance function. What are the
basic objectives of decision -making in corporate finance?
15. Discuss the functions of a Chief Financial Officer.
Multiple Choice Questions
1)The investment decisions should aim at investment in ass ets
only when they are expected to earn a return greater than a
minimum acceptable return is termed as …………………….
a)Interest rate c)growth rate
b)Hurdle rate d)internal rate of return
(2) The traditional view o f financial management looks at :
a)Arrangement of short -term and long -term funds from
financial institutions.
b)Mobilization of funds through financial instruments.
c)Orientation of finance functions with accounting function.
d)All of the above
3)The modern approach to financia lm a n a g e m e n tv i e w :
a) the total funds requirement of the firm
b) the assets to be acquired
c) the pattern of financing the assets.
d) All of the above
4)The financing of long -term assets should be made from:
a) Short -term fund c)long-term funds
b) Debt funds d)equity funds
5)In fund raising decisions, one should keep in view:
a)Cost of various funds and financial risk.
b)Advantages and disadvantages of debt component in capital
mix.
c)Impact of taxation on EPS
d)All of the above .munotes.in

Page 16

166)The financial health of the firm depends on its ability to generate
sufficient _____ to pay its employees, suppliers, creditors and
owners:
a)Profit c)growth
b)Cash d)wealth
7)Liquidity and profitability are _______ goals for the Fin ance
manager.
a)Different c)competing
b)Separate d)finance
8)Wealth maximization means maximizing the ____ of a course of
action.
a)NPV c)profit
b)IRR d)growth
9)_________ maximization objective considers the risk and time
valueo fm o n e y .
a)Profit c)value
b)Wealth d)growth



munotes.in

Page 17

17Unit-2
TIME VALUE OF MONEY
Unit Structure:
2.0 Objectives
2.1 Introduction
2.2 Time Value of Money
2.3 Basic Concepts
2.4 Time Value of Money Relationship
2.5 Future Value of Sing le Amount
2.6 Future Value of Annuity
2.7 Doubling Period
2.8 Present Value of an Uneven Series of Payments
2.9 Present Value of Annuity
2.10 Net Present Value
2.11 Mathematical Tables
2.12 Bond Valuation
2.13Exercise
2.0 OBJECTIVES
After going through this chapter, you will able to:
Understand the concept of time value of money
Compute the time value of money
Calculate the future value as well as the present value of money
Understand the concept of present value and future value of
annuities
2.1 INTRODUCTION
In our economics life, money is not free. Money has time value.
Interest rates give money its time value. If the investor has some
spare cash or funds, he can invest it in savings deposit in a bank
and receive more money later. If the inves tor wants to borrow
money, he must repay a larger amount in the future due to interest.
The result is that Rs. 100 in hand today, is worth more than Rs. 100
to be received a year from now. This is because Rs. 100 today can
be invested to provide Rs. 100 pl us interest after a year. The
interest rates in the economy provide money with its time value.munotes.in

Page 18

18There are two types of decisions which require some consideration
of time value. The first decision involves investing money now in
order to receive future cash benefits. The other decision involves
borrowing now to take current expenditure at a cost of having less
money in the future. The intelligent investor requires familiarity with
the concepts of compound interest.
2.2 TIME VALUE OF MONEY:
In the world of f inance and investment, time does have a
value, Rs. 100 today are more valuable than Rs. 100 a year later.
This is because capital can be employed productively to generate
positive returns. Again, individuals normally prefer current
consumption to future co nsumption. Even in case of inflation, Rs.
100 today represents greater real purchasing power as compared
toRs. 100 one year later. The longer the term of a loan, the greater
will be the amount that must be paid due to interest. Bonds are
worthless to an i nvestor, if the maturity is longer. Therefore, this
makes sense under the general framework of the time value of
money.
2.3 BASIC CONCEPTS:
a)PRESENT VALUE: A present value is the discounted value of
one or more future cash flows.
b)FUTURE VALUE: Afuture value is the compounded value of a
present value.
c)DISCOUNT FACTOR: The discount factor is the present value
of a rupee received in the future.
d)COMPOUNDING FACTOR: The compounding factor is the
future value of a rupee.
Discount and compoun ding factors are functions of two
things: (i) the interest rate used, and (ii) the time between the
present value and the future value. The discount factor decreases
as time increases. The discount factor also decreases as interest
rate increases.
2.4 TIM E VALUE OF MONEY RELATIONSHIP:
The basic time value of money relationships are presented in
the following equations:
(1) PV = FV x DF
(2) FV = PV XCFmunotes.in

Page 19

19Whereas, PV = Present value
FV = Future value
DF = Discounting factor =1
1RtCF = Compounding factor = (1 + R )t
R=R a t eo fi n t e r e s t
T = time in years.
2.5 FUTURE VALUE OF SINGLE AMOUNT:
The future value of an amount invested or borrowed at a
given rate of interest can be calculated if the maturity period is
given. Suppose, a deposi t of Rs. 5,000 gets 10 percent interest
compounded annually for a period of 3 years, the future value will
be:
PV X CF = 5,000 (1.10)3 = 5,000 x 1.331 = Rs. 6,655.
Illustration 1:
MrShashikant deposit ₹1,00,000 with a bank which pays 10
percent interest compounded annually, for a period of 3 years. How
much amount he would get a maturity?
Solution
FV =P VXC F
=1 , 0 0 , 0 0 0x( 1 . 1 0 )3
=1 , 0 0 , 0 0 0x1 . 3 3 1
=₹1,33,100
Mr. Shashikant will get ₹1,33,100 after 3 years.
2.6 FUTURE VALUE OF ANNUITY :
An annuity is a series of payments of a fixed amount for a
specified number of periods. When payments are made at the end
of each year, it is called ordinary annuity. On the other hand when
the payments are ma de at the beginning of the year, it is called an
annuity due. Normally, it is assumed that the first annuity payment
occurs at the end of the first year.11RtFVa AR
Where A = Periodic cash payments
R=A n n u a li n t e r e s tr a t e
T=t i m ei ny e a rs / duration of annuity
The value of11RtRcan be determined by using the Time value
of money tables.munotes.in

Page 20

20The Future value Interest Factors (FVIFA) for various years
are a shown in table:
Illustration 2:
Four equal annual payments of Rs. 5,000 are made into a deposit
account that pays 8 percent interest per year. What is the future
value of this annuity at the end of 4 years?
Solution
The future value of annuity11RtFVa AR
=₹5,000 x FVIFA @ 8%
=₹5,000 x 4.5061
=₹22530.50.
2.7 DOUBLING PERIOD
Sometimes, investor should know how long it will take to
double his money at a given rate of interest. In this case, a rule of
thumb called the rule of 72, can b e used. This rule works pretty well
for most of the interest rates. The rule of 72 says that it will take
seventy -two years to double your money at 1 percent interest. You
can calculate the doubling by dividing 72 by the interest rate. You
can also estimat e the interest rate required to double your money in
the given number of years by dividing number of years into 72.
For example, if the interest rate is 12 percent, it will take 6
years to double your money (72+23). On the other hand, if you
want to doubl e your money in 6 years, the interest rate should be
12 percent (72+6).munotes.in

Page 21

21A more accurate method used for doubling your money is
using the rule of 69. According to this rule, the doubling period of an
investment is = 0.35 + 69 Thus the doubling period of Int erest rate
investment of different rates of interest can be determined as
follows :
As per rule of 69, the doubling period will be
1)Interest rate 12%690.35 0.35 5.75 6.112years
2)Interest rate 15%690.35 0.35 4.60 4.9515years
Illust ration 3 :
If the interest rate is 10%, what isthe doubling periods of an
investment at this rate?
Solution
a)As per rule of 72, the doubling period will be
727.210years
b)As per the rule of 69, the doubling period will be690.35 0.35 6.9 7.2515years
PRESENT VALUE:
Many times, investors like to know the present value which
grows to a given future value. Suppose you want to save some
money from your salary to bu ya scooter after 5 years. You should
know how much money sho uld be put into bank now in order to get
the future value after 5 years. The present value is simply the
inverse of compounding used in determining future value. The
general relationship between future value and present value is
given in the following form ula:
11PV FV DF FVR
Illustration 4 :
Find the present value of ₹50,000 to be received at the end of four
years at 12 percent interest compounding quarterly.
PV=FV 1
PV=FV x PVIF at 12%
=`50,000 x 0.623
=`31,150munotes.in

Page 22

222.8 PRESENT VALUE OF AN UNEVEN SERIES OF
PAYMENTS :
The annuity includes the constant am ount in which cash
flows are identical in every period. Many financial decisions involve
constant cash -flow, however, some important decisions are
concerned with uneven cash flows. For example, investment in
shares is expected to pay an increasing series o fd i v i d e n d so v e r
time. The capital budgeting projects also do not normally provide
constant cash flows.
In order to deal with uneven payment streams, we have to
multiply each payment by the appropriate PVIF and then sum these
products to obtain the pres ent value of an uneven series of
payments.
Illustration 5:
MrShah has invested `50,000 on Xerox machine on 1stJan.
2002. He estimates net cash income from Xerox machine in next 5
years as under.
At the end of 5thyear Machine will be sold at Scarp value of
`5,000. Advice him whether his project to viable, considering
interest rate of 10% p.a.
Solution:
Calculation of Present Value of Future Cash Flows :
Note :It is assumed that the net cash income is received at the end
of the year.munotes.in

Page 23

23Considering 10% interest rate, the net present value of all
future cash flows is `75,635 which is higher than present net cash
flow of `50,000. Thus, the project is viable.
2.9 PRESENT VALUE OF ANNUITY :
Many times investors want to know the pres ent value which
must be invested today in order to provide an annuity for several
future periods. For example, A grandfather wants to deposit enough
money today to meet the tuition fees of his grand -son for the next
three years. The interest rate is 8%. Th e present value of this
annuity is the sum of the present values of all the future inflow of
the annuities. The present value of an annuity can be expressed in
the following formula:


11 1111 2 1 111
1PVA ARR RRt
RR t    

Where PVA1 = Present value of an annuity with a duration of ‘t’
periods
A = Constant periodic flow
R = Interest Rate
The present value interest factors for an annuity (PVIF) can
be determined by using the Time Value of Money Tables. The
(PVIF) for various years are given below :
Forall positive interest rates, PVIFA for the present value of
an Annuity is always less than the number of periods the annuitymunotes.in

Page 24

24runs, whereas FVIFA for the future value of an annuity is equal to
or greater than the number of periods.
Illustration 6 :
What i st h ep r e s e n tv a l u eo fa4y e a r sa n n u i t yo f `8,000 at
12% interest?
Solution:111RtPVA ARR t
The value of111RtRR tas per table is 3.0373
=₹8,000 x PVIF at 12%
=₹8,000 X 3.0373
=₹24,298
2.10 NET PRESENT VALUE :
Net Present Value (NPV) is the most suitable method used
for evaluating the capital investment projects. Under this method,
cash inflow and outflows a ssociated with each project are worked
out. The present value of cash inflows is calculated by discounting
the cash flows at the rate of return acceptable to the management.
The cash outflows represent the investment and commitments of
cash in the project at various points of time. It is generally
determined on the basis of cost of capital suitably adjusted to allow
for the risk element involved in the project. The working capital is
taken as a cash outflow in the initial year. The cash inflow
represents th e net profit after tax but before depreciation. A
depreciation is a non -cash expenditure hence it is added back to
the net profit after tax in order to determine the cash inflows. The
Net Present Value of cash inflows and the present value of cash
outflows . If the NPV is positive the project is accepted, and if it is
negative, the project is rejected.
Discounted cash flow is an evaluation of the future net cash
flows generated by a project. This method considers the time value
of money concept and hence it is considered better for evaluation
of investment proposals. If these are mutually exclusive projects,
this method is more useful. The Net Present Value is determined as
follows :
NPV = Present value of future cash inflows –Present value of cash
outflo ws.
Illustration 7:
An investment of `40,000 made on 1/4/2002 provides inflows as
follows:munotes.in

Page 25

25
Which alternative would you prefer in the investor’s expected return
is 10%? Give reason(s) for your preference.
Solution
Calculation of Present Values:
Alter native I
Alternative II
The net present value of all future cash flows is `40,789 in
case of ‘alternative I ’and `39,962 in case of ‘alternative II’.T h e
NPV in case of ‘alternative I’is higher at 10% discounting factor.
Hence, ‘alternative I ’is preferred for investment.
Illustration 8 :
A Finance company has introduced a scheme of investment of `
40,000. The returns would be `8,000, `10,000, `11,000 and `
12,000 in the next five years. The indicated rate of interest is 10%
Compute the p resent value of the investment and advise regarding
the investment.munotes.in

Page 26

26Solution :
i)Present value of investment = `40,000.
ii)Present value of returns :
iii)Present value of investment is `37,188 which is lower than
investment of `40,000. The ne t present value (i.e. 37,188 -40,000
=`2,812) is negative. Hence the investment is not profitable at
10% interest.
Illustration 9:
The share of Ridhi Ltd ( F.V. of `10) was quot edat`102 on
01.04.2002 and the price rose to `132 on 01.04.2005. Divi dends
were received at 10% on 30thJune each year. Cost of funds was
10%. Is it a worth -while investment, considering the time value of
money? (Present value factor at 10% were 0.909, 0.826 and 0.751)
Solution
Calculation o fP r e s e n tV a l u eo fC a s hi n f l o w s :
Considering the time value of money, the NPV is negative,
hence, it is not a wise investment.munotes.in

Page 27

27Illustration 10 :
XYZ & Co. is considering investing in a project requiring a capital
outlay of `2,00,000 . Forecast for annual income after tax is as
follows:
Evaluate the project on the basis of Net Present Value taking 14%
discounting factor and advise whether XYZ & Co. should invest in
the project or not ? The Present value of Re. 1 at 14% discounting
rate are 0.8772, 0.7695, 0.6750, 0.5921 and 0. 5194.
Depreciation = 20% of 2,00,000 = `40,000
Profit after tax is given.
The cash inflow after tax (CFAT) = Profit After Tax (PAT +
Depreciation.
Net Present Value is positive; hence XYZ & Co should invest
in the project.
Illustration 11:
Find out the present value of a debenture from the following :
Face value of debenture ₹1,000
Annual Interest Rate 15%
Expected return 12%
Maturity Period 5y e a r smunotes.in

Page 28

28(Present values of Re. 1 at 12% are, 0.8929, 0.7972, 0.7118,
0.6355 and 0.5674)
Solution
PV =I(PVAF) + F (DF)
=I(PVAF 12% for 5 years) + F (DF 12% for 5 years)
=150 (3,6048) + 1,000 (0.5674)
=`540.72 + 567.40
=`1108.12
Illustration 12 :
MrVishwanathan is planning to buy a machine which would
generate cash flow as follo ws:
If discount rate is 10%, is it worth to invest in machine?
Solution :
Calculation of Net Present Value
As the NPV is positive, it is worth investing in the machine.
Illustration 1 3:
A machine cost `80,000 and is expected to produce the following
cash flows :munotes.in

Page 29

29
If the cost of capital is 12 percent, is it worth buying the
machine?
Solution:
Calculation of Net Present Value
As the Net Present Value is negative, it is not worth buying
the machine.
Illustration 14 :
Find the compounded v alue of annuity where three equal yearly
payments of `2000 are deposited into an account that yields 7%
compound interest.
Solution
The future value of annuity FVa = A11RtR=`2,000 (FVAFA @ 7% for 3
years)
=`2,000 x 3.21 5
=`6,430
Illustration 15:
Calculate the compound value when `10,000 are invested for 3
years and the interest on it is compounded at 10% p.a. semi
annually.
FV = PV x CF
FV = PV x (1 + R)t
=10,000 X1012 32
=10,000 (1.05) 6
=`10,000 x 1.340
=`13,400munotes.in

Page 30

302.11 MATHEMATICAL TABLES
Tale A -1Present Value of Re. 1 :
11PVIFKn
munotes.in

Page 31

31Table A -1( c o n t i n u e d )
munotes.in

Page 32

32Table A -2Present Value of an Annuity of Re. 1 per period for n
periods.

11
111kkn
PVIFk

munotes.in

Page 33

33Table A -2( c o n t i n u e d )
munotes.in

Page 34

34Table A -3 Future Value of Re. 1 at the end of n Periods.
PVIF = (1 + k) n
munotes.in

Page 35

35Table A -3( c o n t i n u e d )
munotes.in

Page 36

36Table A -4Sum of an Annuity of Re. 1 per period of n Periods :
111
1n n
k tkPVIFAk

munotes.in

Page 37

37Table A -4( c o n t i n u e d )
2.13 BONDS VALUATION
What is Bond?
Bonds are financial instrument which represents the
borrowings of the issuing authority and pays a fixed amount of
interest at a rate specified at the time of issue. From an investor s
point of view it is a fixed return earning instruments.munotes.in

Page 38

38Terms related to Bonds
a.Principal Value or Face Value: It is the nominal or principal
value of the Bond. It is the price printed on the face of the
Bonds issued by the company.
b.Interest rate or Coupo n rate: A coupon rate is the specific
interest rate which is paid at specific intervals to the bond
holders.
c.Maturity Period: It the total time period for which a bond is
issued.
d.Net Proceeds: It is the total amount of funds raised by the
company by the is sue of the bonds.
e.Redemption: It is the repayment of the amount to the bond
holders at the time of maturity.
f.Redemption Value: It is the total amount that is paid to the
holders of the bond at the time of maturity.
Valuation of Bonds:
The bond is to be va lued based on the present value of the
expected cash inflows from such bonds in the form of Principal
amount repayment and interest received over the life time of the
bonds.
V=I (PVIFA r, n) +P(PVIF r, n)
Where,
V=value of the bond
I=Annual i nterest payable on the bond
P=Principal amount of the bond repayable at the time of
maturity (at Par/Premium/Discount)
r=Discount rate or expected rate of return
n=maturity period of the bond.
PVIFA = Present Value Annuity Factor
PVIF = Present Value Interest Factor
Alternatively one can use table to find the value of the Bonds:
Statement showing Valuation of Bond
A B C=AxB
Year (n) Cash Inflow DF @ r%
1 XX (I) X XXX
2 XX (I) X XXX
3 XX (I) X XXX
4 XX (I) X XXX
5 XX (P + I) X XXX
Value of the Bond (Sum of Column ‘C’) XXXmunotes.in

Page 39

39I = Annual interest payable on the bond
P = Principal amount of the bond repayable at the time of maturity
(at Par/Premium/Discount)
r = Discount rate or expected rate of return
n=m a t u r i t yp e r i o do ft h eb o n d .
Illustration 1:
Ab o n dh a v i n gp a rv a l u eo f `100 bears a coupon rate of
14% and has maturity period of 5 years. The required rate of return
on the bond is 12%. What should be the value of the bond?
What will be the answer if the required rate of return if?
a. 12%
b. 14%
b. 16%
Solution:
a. V=I( P V I F A r,n)+P( P V I F r,n)
V=1 4 ( 3 . 6 0 4 8 )+100(0.5674)
V=5 0 . 4 7+5 6 . 7 4
V=1 0 7 . 2 1
OR
a.
Year Cash Inflow DF @ 12%
1 14 0.8929 12.50
2 14 0.7972 11.16
3 14 0.7118 9.97
4 14 0.6355 8.90
5 114 (100 + 14) 0.5674 64.68
Value of the Bond 107.21
b. V=I( P V I F A r,n)+P( P V I F r,n)
V=1 4 ( 3 . 4 3 3 2 )+100(0.5194)
V=4 8 . 0 6+ 51.94
V=1 0 0 . 0 0
ORmunotes.in

Page 40

40b.
Year Cash Inflow DF @ 14%
1 14 0.8772 12.28
2 14 0.7695 10.77
3 14 0.6750 9.45
4 14 0.5921 8.29
5 114 (100 + 14) 0.519 4 59.21
Value of the Bond 100.00
c. V=I (PVIFA r,n)+P (PVIF r,n)
V=1 4 ( 3 . 2 7 4 4 ) +100(0.5194)
V=4 5 . 8 4+4 7 . 6 1
V=9 3 . 4 5
OR
c.
Year Cash Inflow DF @ 16%
1 14 0.8621 12.07
2 14 0.7432 10.40
3 14 0.6407 8.97
4 14 0.5523 7.73
5 114 (100 + 14) 0.4761 54.28
Value of the Bond 93.45
Illustration 2:
Lion Ltd. has issued a debenture with face value of `100
bearing coupon @ 10% p.a. maturing after 6 years at par. The
expected rate of return of investor is 15%. Should investor buy the
debentures if the current market price of debenture is `85?
(TYBAF, May 2016 (Adapted))munotes.in

Page 41

41Solution:
Statement sho wing Valuation of Bond:
Year Cash Inflow DF @ 15% Amount
1 10 0.8696 8.70
2 10 0.7561 7.56
3 10 0.6575 6.58
4 10 0.5718 5.72
5 10 0.4972 4.97
6 110 (100 + 10) 0.4323 47.55
Value of the Debentures 81.08
The value of the Bond is 81.08 and it is priced at ₹85 in the
market, so it is overpriced and ther efore the investor is advised not
to buy the debentures .
Illustration 3
Darshan Ltd. wants to issue debentures redeemable after 7
years at a premium of 10%. Face value of debentures is `1,000.
The company proposes to issue so as to yield a return of 12% p.a.
to the investor. The coupon rate for the first three years will be 13%
p.a. which will be increased by 2% p.a. for the remaining life. As
CFO of the company advice the issue price of the debenture.
(TYBAF,Nov 2016)
Solution:
Statement showing Valuation of Bond
Year Cash Inflow DF @ 12%
1 130 0.8929 116.08
2 130 0.7972 103.64
3 130 0.7118 92.53
4 150 0.6355 95.33
5 150 0.5674 85.11
6 150 0.5066 75.99
7 1,250 (1,100 + 150) 0.4523 565.38
Value of the Bond 1,134.05munotes.in

Page 42

42Redemption at Premium: 1,000 + 100 (1,000 x 10%
Premium.)
Kindly Note:
1. When the interest is fl uctuating, we should use Table based
format.
Illustration 4:
Sanjana Ltd. has issued bonds with face value of `1,000
bearing interest @ 24% p.a. payable half yearly maturing after 5
years at par. The expected rate of return of an investor is 12%.
Should the investor buy the bonds if the current price of bond listed
in the market is `1,000? (Adapted TYBAF Nov. 2016).
Solution:
Statement showing Valuation of Bond
Year Cash Inflow DF @ 6**%
1 120* 0.9434 113.21
2 120 0.8900 106.80
3 120 0.8396 100.75
4 120 0.7921 95.05
5 120 0.7473 89.68
6 120 0.7050 84.60
7 120 0.6651 79.81
8 120 0.6274 75.29
9 120 0.5919 71.03
10 1,120 (1,000 + 120) 0.5584 625.41
Value of the Bond 1,441.62
* Coupon Rate: 24% p.a. therefore 12% for 6 months
** Interest Rate: DF @ 12% p.a.
Illustration 5:
Credit unlimited Ltd. has issued fully convertible bonds with
face value of `100 with coupo n rate of 16% p.a. which will convert
in 10 equity shares of `10 each at the end of 6 years. Find out the
value of debentures if the expected rate of return of an investor is
20% p.a. and expected market price of one share after 6 years is
`28.50. Interest on debentures will be paid on half yearly basis.
(TYBAF Nov. 2016)munotes.in

Page 43

43Solution:
Statement showing Valuation of Bond
Year Cash Inflow DF @ 10%
1 8 0.9091 7.27
2 8 0.8264 6.61
3 8 0.7513 6.01
4 8 0.6830 5.46
5 8 0.6209 4.97
6 8 0.5645 4.52
7 8 0.5132 4.11
8 8 0.4665 3.73
9 8 0.4241 3.39
10 8 0.3855 3.08
11 8 0.3505 2.80
12 293* 0.3186 93.35
Value of the Bond 145.31
*Cash Inflow in Last Year = (10 Shares x `28.50(Converted Value
of Debentures + 8)
Yield to Maturity :
Yield to Maturity (YTM) (alternatively referred as redemption or
book yield )is the specula tiverate of return or interest rate of a
fixed -rate security, such as a bond . The YTM is based on the belief
or understanding that an investor purchases the security at the
current market price and holds it until the security has matured
(reached its full value), and that all interest and coupon payments
aremade in a timely fashion. In simple words ‘Yield to Maturity’ is
the rate of return, mostly annualised, that an investor can expect to
earn if they hold the bond till maturity.
Where,
I = Interest
R.V. = Redemption Value
N.P. = Net Proceeds/Market Value
n = no of years/periodsmunotes.in

Page 44

44Illustration 1:
Cairo Ltd.’s bond with a par value of `500 is currently traded at `
435. The coupon rate is 12% and it has a maturity period of 7
years. What is yield to maturity?
Solution:
YTM =
YTM = 14.82%
Illustration 2:
What is YTM of each bond? Which bond will you recommend for
investment?
Bond Coupon
RateMaturity Price/ `100 Par Value
Bond X 11% 10 years `76
Bond Y 12% 7y e a r s `69
(TYBAF Nov. 2019)
Bond X Bond Y
.. .... . 1 0 0.. ..
2RV N PInterestnYTMRV N P                  100 761110100100 76
2
13.41008815.23      
100 69127100100 69
2
16.4310084.519.44%      
Duration of Bond
The concept of duration is straightforward. Duration is
nothing but the average time taken by an investor to collect his/her
investment. If an investor receives a part of his/her in vestment over
thetime on specific intervals before maturity, the investment will
offer him the duration which would be lesser than the maturity of
the instrument. Higher the coupon rate, lesser would be the
duration.
It measures how quickly a bond will r epay its true cost. The
longer the time it takes the greater exposure the bond has to
changes in the interest rate environment.munotes.in

Page 45

45Illustration 3:
Calculate the duration of Bond from the following details.
Face Value = `1,000
Coupon Rate (payable annually) =1 3%
Years to Maturity = 5 years
Redemption value = `1,000
Current Market Price = `1036
Yield toMaturity = 12% ( TYBAF. Apr. 2019).
Statement showing calculation of Duration of Bond
1 2 3 4=2x3 5=1x4
Year Interest @ 13%YTM @
12%PVCFYear x
PVCF
1 130 0.8929 116.08 116.08
2 130 0.7972 103.64 207.28
3 130 0.7118 92.53 277.59
4 130 0.6355 82.62 330.48
5 1130 0.5674 641.16 3,205.80
1,036.03 4,137.23
Duration of Bond = ∑ Year x PVCF
∑ PVCF
= 4,137.23
1,036.03
Duration of Bond = 3.99 Years
Illustration 4:
The following data is available for a bond. Face value is `100,
Coupon rate is 14%, years to maturity is 5 years, and redemption
value is `100. YTM is l5%. Calculate duration of bond.
(TYBAF, Nov. 2019)
Statement showing calculation of Duration of Bond
1 2 3 4=2x3 5=1x4
Year Interest @ 14%DF @
15%PVCFYear x
PVCF
1 14 0.8696 12.17 12.17
2 14 0.7561 10.59 21.17
3 14 0.6575 9.21 27.62
4 14 0.5718 8.01 32.02
5 114 0.4972 56.68 283.40
96.65 376.39munotes.in

Page 46

46Duration of
Bond=∑ Year x
PVCF
∑ PVCF
376.39
96.65
Duration of
Bond= 3.89 Years
2.13EXERCISES
1.What do you understand by time value of money?
2.What are the possible reasons that must have time value
despite not being put to use?
3.What do you understand by future value and present value of
money?
4.What are annuities? And why such values are calculated/
5.How do you determine the equated monthly installments?
2)Indicate the right answer with your reasoning:
a)Which provides money its time value?
i)Investment
ii)Interest Rates
iii)Market Rates
iv)Call Rates
b)In approximately, how many years would you expect to
double your money at 8% per annum?
i)8y e a r s
ii)12 years
iii)9y e a r s
iv)10 years
c)When payments are made at the end of each year, it is
known as ________ annuity.
i)Annuity due
ii)Ordinary annuity
iii)Perpetuity
iv)Fixed annuitymunotes.in

Page 47

47d)When compounding is done more frequently than annually,
the effective rate of inter est is ___________.
i)greater than the nominal rate of interest.
ii)lower than the normal rate of interest.
iii)equal to nominal rate of interest.
v)normal
Hint (Ans (a) –ii, (b) –iii, (c) –ii (d) –i)
2)Ramesh deposited `2,000 for 3 years period at 12% interest
which is credited at the end of every six months. What will be
the total amount credited to Ramesh’s Account at the end of 3
years?
3)Mohan plan to send his son for higher studies in America after 5
years. He expects the cost of the study to be `4,00,000. How
much should he save annually to have a sum of `4,00,000 at
the end of 5 years. If the interest rate is 9%
4)A bank promises to give you `5,000 after 10 years in exchange
of`2,000 today. What is the interes tr a t ei n v o l v e di nt h i so f f e r ?
5)Mukesh deposits `2,00,000 in Saraswati Co -op Bank which
pays 10 per cent interest. How much he withdraw annually for a
period of 15 years?
6)Avinash wants to invest @ 8% p.a. compound interest, such
amount as will am ount to `50,000 at the end of three years.
How much should he invest? (Ans. 39,642)
7)A company has advertised for deposits from the public. If you
deposit `1,000 now, you would receive `1,464 at the end of 4
years or `1,611 at the end of 5 years . What rates of interest is
the company paying? (Ans. 10%)
8)Four equal annual payments of `4000 are made into a deposit
account that pays and per cent interest per year. What would be
the future value of this annuity at the end of 6 years?
(Ans. `29,342)
9)You can save `20,000 a year for 5 years and `9,000 and `
3,000 a year for 10 years thereafter. What will these saving
cumulate to al the end of 15 years if the rate of interest is 10
percent? (Ans. 1,69,913)
10)What is the present value of the following cash stream if the
discount rate is 12%?munotes.in

Page 48

48
(Ans. `31,479)
11)Find out the present value of debenture from the following :
Face Value of debentures `1000
Annual interest rate 12%
Expected Return 10%
Maturity period 5 years
12)ABank advertise that it will pay a lump sum of `45,740 at the
end of 8 years to the investors who deposit annually `4,000 for
8 years what is the interest rate bank is paying?

munotes.in

Page 49

49Unit-3
RATIO ANALYSIS
Unit Structure :
3.0 Objectives
3.1 Introduction
3.2 Ratio Meaning and Definition
3.3 Importance of Ratio Analysis
3.4 Advantages of Ratio Analysis and Limitations of Ratio
Analysis
3.5 Classification of Ratios
3.6 Liquidity Ratios / Short Term Solving Ratios
3.7 Leverage Ratios / Capital Structure Ratios
3.8 Asset Management Ratio / Turnover Ratio / Performance
Ratio / Activity Ratio
3.9 Profitability Ratios
3.10 Operating Ratios
3.11 Market based Ratios
3.12 Ques tions
3.0 OBJECTIVES
After studying the unit the students will be able to
understand:
Meaning and definition of Ratio.
Advantages of preparing Ratio.
Importance of Ration analysis.
Types of ratios.
Computation of operating ratio.
Ratios measuring short -term solvency and liquidity of the firm.
Ratios indicating long -term financial stability of the firm.
Ratios indicating debt service capacity.
Analysis of profitable team ratio.
Ratios indicating the efficiency of the firm.
Limitation of Ratio analysis.munotes.in

Page 50

503.1 INTRODUCTION
Ratio analysis allow the interested parties like Shareholders,
investors, creditors, government, bankers to come to a conclusion
about the company’s performance. The appraisal of the ratio will
make proper analysis about the strength and weakness of the
firms. The calculation of ratio is an easy and simple task, but the
proper analysis and interpretation can be made only by expose.
While interpreting the financial data the analysis has to be careful in
the limitations imposed by the accoun ting concepts and methods.
Ratio analysis is a helpful in providing valuable insight when a
company is in financial position.
3.2 RATIO MEANING AND DEFINITION
The term ratio is used to describe the relationships between
figures in a balance sheet or in a profit and loss account or in a
budgetary control system or in any other part of the accounting
organization. The accounting ratio indicates the quantitative
relationship which is used for analysis and decision making. It gives
data for analysis of inter firm and intra firm comparison. A ratio is a
quotient of two numbers and the relation expressed between two
accounting figures is known as `accounting ratio’. Ratio analysis is
a very powerful analysis tools useful for measuring performance of
an organizat ion. The ratio analysis concentrates on the internal
relationship among the figures in financial statement. The analysis
helps the management to make future projections on the basis of
the past records.
Definition
“Ratio is a yardstick used to evaluate th e financial condition
and performance of a firm, relating two pieces of financial data to
each other.”
-James C. Van Harne
“The relation of one amount, a to another b, expressed as
the ratio of a to b”.
-Kohler
“Ratio is a fraction whose numerator is t he antecedent and
denominator the consequent.”
“Ratio is the relationship or proportion that one amount bears
to another, the first number being the numerator and the later
denominator.”
-H.G. Guthmannmunotes.in

Page 51

51From the above definitions, we can clearly say that ,“ r a t i oi s
a relationship between two figures or factors or variables”. This
relationship helps to analyze, interpretation and the financial
condition of the firm. The accounting ratios indicate a quantitative
relationship which is used for analysis and d ecision -making.
3.3 IMPORTANCE OF RATIO ANALYSIS
The major benefits arising from ratio analysis are as follows :
1.Ratio analysis is a very powerful analytical tool, useful for
measuring performance of an organization.
2.Ratio analysis concentrates on the inter -relationship among the
figures appearing in the financial statements.
3.Ratios make comparison easy. The said ratio is compared with
the standard ratio and this shows the degree of efficiency
utilization of assets, etc.
4.The results of tw o companies engaged in the same business
can be easily compared (inter -firm comparison) with the help of
ratio analysis.
5.Short -term liquidity position and long -term solvency position can
be easily ascertained with the help of ratio analysis.
6.Ratio analysis helps the management to analysis the past
performance of the firm and to make further projections.
7.Ratio analysis allows interested parties to make evaluation of
certain aspects of the firm’s performance.
8.Its importance lies in analyzing t he probable casual relationship
between two past results.
9.By effectively using the ratios, one can find out the growth or
decline of an enterprise with the help of them, future actions can
be taken.
10.Ratio analysis helps the management to analyze th e past
performance of the firm and to make further projection.
11.Ratio analysis allows interested parties, like shareholders,
investors, creditors and analysts to make an evaluation of
certain aspects of a firm’s performance.
12.The appraisal of the rat ios will make proper analysis about the
strengths and weaknesses of the firm’s operations.munotes.in

Page 52

523.4 ADVANTAGES OF RATIO ANALYSIS AND
LIMITATIONS OF RATIO ANALYSIS
3.4.1 ADVANTAGES OF RATIO ANALYSIS
Advantages of ratio analysis for shareholders and
prosp ective investors;
1.Shareholders and prospective investors will analyse ratios for
making decisions regarding investment and disinvestment.
2.The credit rating agencies will analyse the ratios of a firm to give
the credit rating to the firm.
3.Gover nment agencies will analyse the ratios of a firm for review
of its performance.
4.Bankers who provide working capital will analyse ratios for
appraising the creditworthiness of the firm.
5.Financial institutions that provide long -term debt will analyse
ratios for project appraisal and debt servicing capacity of the
firm.
6.Financial analysts will analyse ratios for making comparisons
and recommending to the investing public.
3.4.2 LIMITATIONS OF RATIO ANALYSIS
The following limitations of ratio ana lysis must be taken into
account :
1.Over -use of ratios as controls on managers could be
dangerous; in the management might concentrate more or
simply improving the ratio that one dealing with the significant
issues.
2.Ratios can only provide guideline st ot h em a n a g e m e n t .T h e y
are only the means. However, they scratch surfaces and raise
questions. This limitation of ratios may force the management to
have detailed investigation of the situation under question.
3.The standards will differ from industry to industry. Comparison
of ratios of firms belonging to different industries is not
suggested.
4.Since ratios are calculated from past records, there are no
indicators of the future.
5.Proper care should be exercised to study only such figures as
have a cause and effect relationship, otherwise, ratios will only
be meaningless or misleading.
6.The reliability and significance attached to ratios depends on the
accuracy of data based on which ratios are calculated.munotes.in

Page 53

537.Ratios of a company can have meani ng only when they are
compared against standard. Past performance of the same
company cannot be benchmarked when there is change in
circumstances.
8.The change in price levels due to inflation will distort the
reliability of ratio analysis.
9.The analy st should have through knowledge of methods of
window -dressing.
10.Single accounting ratio is not useful at all, unless it is studied
with other accounting ratios. This limitation of ratios
necessitates inter -firm and intra -firm comparisons.
11.Ratios ar eb a s e do n l yo nt h eq u a n t i t a t i v ei n f o r m a t i o n .H e n c e ,
qualitative information (i.e. character, managerial ability etc.)
places a limit on the ratios.
12.Ratios are subject to arithmetical accuracy of the financial
statements. Moreover, financial statements also include
estimated data like provision for depreciation, for bad and
doubtful debts, etc. Hence, results revealed by ratios are subject
to such estimates.
13.Ratios are computed on the basis of financial statements which
are historical in nature.
14.Knowledge of ratios alone in meaningless unless their
composition is also ascertained.
15.Lack of homogeneity of data, personal judgment, lack of
consistency, etc. is the factors that limit the conclusion to be
derived on the basis of accounting ratios.
16.Rations are calculated form financial statements which are
affected by the financial bases and policies adopted on such
matters as depreciation and the valuation of stocks.
17.A ratio is a comparison of two figures, a numerator and a
denominator. In c omparing ratios, it may be difficult to determine
whether differences are due to changes in the numerator, or in
the denominator or in both.
18.Ratios are interconnected. They should not be treated in
isolation. The effective use of ratios, therefore, dep ends on
being aware of all these limitations and ensuring the following
comparative analysis, they are used as a trigger point for
investigation and corrective action, rather than being treated as
meaningful in them.
19.The analysis of ratios clarifies tr ends and weaknesses in
performance as a guide as long as proper comparisons are
made the reasons for adverse treads or deviations from the
norm are investigated thoroughly.munotes.in

Page 54

5420.While making inter -firm comparison, the analyst must keep in
mind that differen t firms follow different accounting policies, e.g.,
depreciation allowance, valuation of inventory, etc.
3.5 CLASSIFICATION OF RATIOS
a.Calculation according to statement of sources and income
statement,
b.Classification according to functions / objec tives.
c.Classification according to users,
d.Classification according to time, and
e.Classification according to nature or importance in financial
analysis.
The above classifications can be elaborated as follows:
CLASSIFICATION ACCORDING TO STATE MENT OF
SOURCES AND INCOME STATEMENT
i)Balance sheet Ratios :Where both numerators and
denominator figures are taken from he balance sheet, such as
current ratio, quick ratio, proprietary ratio, etc.
ii)Profit and Loss (Revenue) Ratios: Ratios calculate df r o m the
figures in the profit and loss account. At times they are also
called operating ratios. Some of them are the profit and loss
ratios, gross profit ratio, net profit ratio, expenses ratio, etc.
iii)Position -cum-Revenue ratios :Where ratios are c alculated by
taking one figure from Balance sheet and another from Profit
and Loss Account, they are known as combined or consolidated
or complex or position -cum-revenue ratios. For example, return
on proprietor’s fund, return on capital employed, turnover of
debtors and creditors separately, earnings ratio, etc.
CLASSIFICATION ACCORDING TO FUNCTIONS / OBJECTIVES
i)Financial ratios :Include solvency and liquidity ratios.
ii)Profitability ratios :For example, gross profit ratio, net profit
ratio, operati ng ratio, return on capital employed, expense
rations, etc.
iii)Turnover or Activity ratios :For example, turnover of stock,
turnover or debtors, turnover of creditors etc.
iv)Market Test Ratios :For example, Earnings per share (EPS),
price earning rat io,and dividend yield ratio, equity dividend
cover, net cash inflow, book value per share, etc.munotes.in

Page 55

55CLASSIFICATION ACCORDING TO USERS
In view of the requirements of the various uses of ratios, we
may classify them into the following four important categor ies.
i)Liquidity ratios
ii)Leverage ratios
iii)Activity ratios
iv)Profitability ratios.
CLASSIFICATION ACCORDING TO TIME
i)Structural ratios: It means the ratios computed from data
referring to the same point of time, e.g., ratios of a particula r
month or year.
ii)Trend ratios: It means the ratios computed between the items
referred to different periods of time.
CLASSIFICATION ACCORDING TO NATURE OR IMPORTANCE
IN FINANCIAL ANALYSIS
i)Primary ratio :e . g . ,O p e r a t i n gp r o f i tt oC a p i t a le m p l o y e d .
ii)Secondary or supporting ratio: e.g. Stock velocity, Creditors’
velocity, Expenses ratios, etc.
In the above, we have seen different classification of ratios is
available on different basis. From the above let us understand in
detail some of the ve ry important ratios which we use frequently in
different situations of decision -making process, they are on the
basis of users or nature or purpose with which a ratio is calculated.
On the basis of users or the nature or purpose, accounting ratios
may be c lassified into six broad categories:
1.Liquidity Ratios or Short -term Solving Ratios.
2.Capital Structure or Leverage Ratios.
3.Turnover Ratios or Activity Ratios or Performance Ratios or
Asset Management Ratio.
4.Profitability Ratios.
5.Operat ive Ratios.
6.Market based Ratios.
3.6 LIQUIDITY RATIOS / SHORT TERM SOLVING
RATIOS
The liquidity ratios measure the liquidity of the firm and its
ability to meet its maturing short -term obligations. Liquidity is
defined as the ability to realized va lue in money, the most liquid of
assets.
Liquidity refers to the ability to pay in cash, the obligations
that are due. The corporate liquidity has two dimensions viz.,
quantitative and qualitative concepts. The quantitative aspectmunotes.in

Page 56

56includes the quantum, structure and utilization of liquid assets and
in the qualitative aspect, it is the ability to meet all present and
potential demands on cash from any source in a manner that
minimizes cost and maximizes the value of the firm. Thus,
corporate liquidity is a vital factor in business. Excess liquidity,
though a guarantor or solvency would reflect lower profitability,
deterioration in managerial efficiency, important ratios in measuring
short -term solvency are (a) Current ratio, (b) Quick ratio (c)
Absolute l iquid ratio and (d) Defensive Interval ratio.
1.Current Ratio
This ratio measures the solvency of the company in the
short -term; current assets are those assets which can be converted
into cash within a year. Current liabilities and provisions are tho se
liabilities that are payable within a year.
Current Assets, Loans & Advances
Current Liabilities & Provisions
A current ratio of 2:1 indicates a highly solvent position A
current ratio of 1.33: 1 is considered by banks as the minimum
acceptable leve l for providing working capital finance. The
constituents of the current assets are as important as the current
assets themselves for evaluation of a company’s solvency position.
A very high current ratio will have adverse impact on the profitability
of the organization. A high current ratio may be due to the piling up
of inventory, inefficiency in collection of debtors, high balances in
cash and bank accounts without proper investment etc.
Advantages of Current Ratio
1.This ratio indicates the extent of current asset available to meet
the current obligations. It is only from the current assets the
immediate obligations (current liabilities) are met with. Therefore,
the interest of creditors lies in this ratio.
2. The safe ratio is 2:1. This means, for e very current liability of Rs.
1, there should be current assets of Rs. 2, so that the firm can
conveniently meet its current obligations, even if the assets like
stock or debtors are not quickly realized.
3. This margin also leaves sufficient amount as wo rking capital to
carry out day -to-day transactions.
4. This is useful in assessing the solvency and liquidity position of
the company.
2.Quick/Liquid/Acid Test Ratio
Quick ratio is used as a measure of the company’s ability to
meet its current obliga tions. Since bank overdraft is secured by themunotes.in

Page 57

57inventories, the other current assets must be sufficient to meet
other current liabilities.
Current Assets, Loans and Advances –Inventories –Prepaid
Expenses –Advance
Current Liabilities Provisions –Bank o verdraft –Cash Credit
A quick ratio I: I tindicates highly solvent position. This ratio serves
as a supplement to the current ratio in analyzing liquidity.
Advantages of the Quick Ratio
a.This ration is very useful for cross -checking the performance in
other areas of economic management of an enterprise. Thus,
the liquid ratio, cross -checked with inventory throw light on the
inventory accumulation. In addition, the liquid ratio can throw
light on certain other aspects of inventory management, which
willbe pointed out later.
b.It is an improved variant of the current ratio in arriving at a
liquidity index for an enterprise.
3.Absolute Liquid / Super Quick Ratio
It is the ratio of absolute liquid assets to quick liabilities.
However, for calculati on purposes, it is taken as ratio of absolute
liquid assets to current liabilities. Absolute liquid assets include
cash in hand, cash at bank and short -term or temporary
investments. The formula is
Absolute Liquid Assets
Current Liabilities
Absolute L iquid Assets –Cash in hand + Cash at Bank + Short
term investments. The ideal absolute liquid ratio is taken as 1: 2.
4.Defensive –Interval Ratio
A firm’s ability to meet current financial obligation is
dependant on the ability to generate daily cas hr e q u i r e m e n t so ft h e
firm. The defensive -internal ration is a measure of liquidity by
comparing the liquid assets against projected daily cash
requirement. The formula is:
Liquid Assets
Projected Daily Cash Requirement
Where, projected daily cash requi rement = Projected cash
operating expenditure / Number of days in a year (365)
Liquid Assets = Cash + Marketable Securities + Debtorsmunotes.in

Page 58

58Projected cash operating expenditure includes all estimated
cash expenses excluding depreciation. The higher the ratio ,m o r e
safety of short -term liquidity.
3.7 LEVERAGE RATIOS / CAPITAL STRUCTURE
RATIOS
The long -term financial stability of the firm may be
considered as dependent upon its ability to meet all its liabilities,
including those not currently payable. The ratios which are
important in measuring the financial leverage of the company are
as follows:
1. Debt -Equity Ratio
This ratio indicates the relationship between loan funds and
net worth of the company, which is known as `gearing’. If the
proportion of d ebt to equity is low, a company is said to be low
geared, and vice versa. A debt -equity ratio of 2:1 is the norm
accepted by financial institutions for financing of projects. Higher
debt-equity ratio of 3:1 may be permitted for higher capital intensive
industries like petrochemicals, fertilizers, power etc. The higher the
gearing, thehigher volatile the return to the shareholders .The
formula is:
Long Term Debt
Shareholders’ Funds
The use of debt capital has direct implications for the profit
accruing t o the ordinary shareholders and expansion is often
financed in this manner with the objective of increasing the
shareholders rate of return. This objective is achieved only if the
rate earned on the additional funds raised exceeds that payable to
the provi ders of the loan. The shareholders of a highly geared
company reap disproportionate benefits when earnings before
interest and tax increase. This is because interest payable on a
large proportion of total finance remains unchanged. The converse
is also tru e, and a highly geared company is likely to find itself in
severe financial difficulties if it suffers a succession of trading
losses. It is not possible to specify an optimal level of gearing for
companies but, as a general rule, gearing should be low in those
industries where demand is volatile and profits are subject to
fluctuation. A debt -equity ratio which shows a declining trend over
the years is usually taken as a positive sign reflecting on increasing
cash accrual and debt repayment. In fact, one of the indicators of a
unit turning sick is rising debt -equity ratio. Usually in calculating the
ratio, the preference share capital is excluded from debt, but if the
ratio is to show effect of use of fixed interest sources on earnings
available to the share holders then it is to be included. On the other
hand, if the ratio is to examine financial solvency, then preference
shares shall form part of the capital.munotes.in

Page 59

59Advantages / Uses of Debt -Equity Ratio
a.This ratio is a measure of contribution of owners to the business
as compared to long -term creditors.
b.It tests the long -term liquidity or solvency of an organization.
2. Shareholders Equity Ratio
This ratio is calculated as follows:
Shareholders Equity
Total Assets (tangible)
It is assumed that larger the proportion of the shareholders’
equity, the stronger is the financial position of the firm. This ration
will supplement the debt -equity ratio. In this ratio the relationship is
established between the shareholders’ funds and the total assets.
Sharehol ders’ funds represent equity and preference capital plus
reserves and surplus less accumulated losses. A reduction in
shareholders equity signalling the over dependence on outside
sources for long -term financial needs and this carries the risk of
higher le vels of gearing. This ration indicates the degree to which
unsecured creditors are protected against loss in the event of
liquidation.
3. Long -term Debt to Shareholders Net worth Ratio
This ratio is calculated as follows:
Long -term Debt
Shareholders Ne t worth
The ratio compares long -term debt to the net worth of the
firm i.e. the capital and fresh reserves less intangible assets. This
ratio is finer than the debt equity ratio and includes capital which is
invested in fictitious assets like deferred ex penditure and carried
forward losses. This ratio would be of more interest to the
contributories of long -term finance to the firm, as the ratio gives a
factual idea of the assets available to meet the long term liabilities.
4. Capital Gearing Ratio
It ist h ep r o p o r t i o no ff i x e di n t e r e s tb e a r i n gf u n d st oe q u i t y
shareholders funds:
Fixed interest bearing Funds
Equity Shareholder’s Funds
The fixed interest bearing funds include debentures, long
term loans and preference share capital. The equity sharehol ders
funds include equity share capital, reserves and surplus. Capital
gearing ratio indicates the degree of vulnerability of earning
available for equity shareholders. This ratio signals the firm which is
operating on trading on equity. It also indicates the changes inmunotes.in

Page 60

60benefits accruing to equity shareholders by changing the levels of
fixed interest bearing funds in the organization.
Advantages of Capital Gearing Ratio
a.Capital gearing ratio measures the company’s capitalization.
b.This ratio is usef ul to the new investors for making sound
investment decision.
c.Capital gearing ratio shows the claim of owners as against the
claim of lenders and preference share holders.
5. Fixed Assets to Long -Term Funds Ratio
The fixed assets are shown as a prop ortion to long -term
funds as follows:
Fixed Assets
Long -term Funds
This ratio indicates the proportion of long -term funds
deployed in fixed assets. Fixed assets represent the gross fixed
assets minus depreciation provided on this till the date of
calcul ation. Long -term funds include share capital, reserves and
surplus and long -term loans. The higher the ratio indicates the safer
the funds available in case of liquidation. It also indicates the
proportion of long -term funds that is invested in working cap ital.
6. Proprietary Ratio
It expresses the relationship between shareholders’ net
worth and total assets.
Shareholders Net worth
Total Assets
Net worth = Equity share capital + Preference share capital +
Reserves –Fictitious assets.
Total assets = Fixed assets + Current assets –Fictitious assets
Reserves earmarked specifically for a particular purpose
should not be included in calculation of net worth. A high
proprietary ratio is indicative of strong financial position of the
business. The highe r the ratio, the better it is.
Advantages or uses of Proprietary Ratio
It also shows the relation between own fund and borrowed
fund. It shows the amount of proprietors funds invested in the total
assets of the firm.
7. Interest Cover
The interest co verage ratio shows how many times interest
charges are covered by funds that are available for payment of
interest.munotes.in

Page 61

61Profit before interest, Depreciation and Tax
Interest
A very high ratio indicates that the firm is conservative in
using debt and a very l ow ratio indicates excessive use of debt.
Interest cove indicates how many times a company can cover its
current interest payment out of current profits. It gives an indication
of problem in servicing the debt. An interest cover of more than 7
times is reg arded as safe and more than 3 times is desirable. An
interest cover of 2 times is considered reasonable by financial
institutions.
8. Debt Service Coverage Ratio (DSCR)
This ratio is the key indicator to the lender to assess the
extent of ability of the borrower to service the loan in regard to
timely payment of interest and repayment of loan installment. It
indicates whether the business is earning sufficient profits to pay
not only the interest charges, but also the installments due of the
principal am ount. The ratio is calculated as follows :
Profit after taxes + Depreciation + Interest on Loan
Interest on Loan + Loan repayment in a year
A ratio of 2 is considered satisfactory by the financial
institutions. The greater debt service coverage ratio in dicates the
better debt servicing capacity of the organization. By means of cash
flow projection, the borrower should work DSCR for the entire
duration of the loan. This will enable the lender to take correct view
of the borrower’s repayment capacity.
9.Dividend Cover
This ratio indicates the number of times the dividends are
covered by net profit. This highlights the amount retained by a
company for financing of future operations.
a)Preference Dividend Cover
Net profit after tax
Preference Dividend
b)Equity Dividend Cover
Net Profit Tax –Preference Dividend
Equity Dividend
Use of Advantages of Debtor Turnover Ratio
1.This ratio helps to monitor credit and collection policies. It can
signal the need for corrective action particularly if compared with
an o r m .munotes.in

Page 62

622.This ratio highlights the impact of management policies on the
liquidity of the enterprise as well as its profitability. It is a
barometer of the general state of health of an enterprise.
3.It is easy to understand, particularly when e xpressed as debtors’
collection period.
3.8 ASSET MANAGEMENT RATIO / TURNOVER
RATIO / PERFORMANCE RATIO / ACTIVITY RATIO
Asset management ratios measure how effectively the firm
employs its resources. These ratios are also called `activity or
turnover ratios’ which involve comparison between the level of
sales and investment in various accounts –inventories, debtors,
fixed assets, etc. Asset management ratios are used to measure
the speed with which various accounts are converted into sales or
cash. Th e following asset management ratios are calculated for
analysis. These ratios also analyse the use of resources and the
utility of each component of total assets. The profitability of the firm
can be determined by activity ratios coupled with the degree of
leverage.
1.Inventory Turnover Ratio
A considerable amount of a company’s capital may be tied up in the
financing of raw materials, work -in-progress and finished goods. It
is important to ensure that the level of stocks is kept as low as
possible, cons istent with the need to fulfil customers’ orders in time.
The ratio is calculated as:
Cost of Goods Sold SalesORAverage Inventory Average InventoryAverage Inventory = (opening stock + closing stock)/2
The higher the stock turnover rate or the lower the stock
turnover period the bette r, although the ratios will vary between
companies. For example, the stock turnover rate in a food retailing
company must be higher than the rate in a manufacturing concern.
The inventory turnover ratio measures how many times a
company’s inventory has bee n sold during the year. If the inventory
turnover ratio has decreased from past, it means that either
inventory is growing or sales are dropping. In addition to that, if a
firm has a turnover that is slower than for its industry, then there
may be obsolete goods on hand, or inventory stocks may be high.
Low inventory turnover has impact on the liquidity of the business.munotes.in

Page 63

632.Inventory Ratio
The level of inventory in a company may be assessed by the
use of the inventory ratio, which measures how much has been tied
up in inventory. The formula is:
Inventory100Current Assets
3.Debtors Turnover Ratio
Debtor’s turnover, which measures whether the amount or
resources tied up in debtors, is reasonable and whether the
company has been efficient in convert ing debtors into cash. The
formula is:
Credit Sales
Average Debtors
The higher the ratio, the better the position.
4. Debtors Collection period or Debtors Velocity Ratio
Average debtors collection period measures how long it
takes to collect amounts from
Average Debtors365Credit Salesin days
The actual collection period can be compared with the stated
credit terms of the company. If it is longer than those terms, then
this indicates inefficiency in collecting debts.
5. Bad Debts to Sales Ratio
It measur es the proportion of bad debts to sales and
calculated as:100SalesBad Debts
Bad debts to sales ratio indicate the efficiency of the credit
control procedures of the company. Its level will depend on the type
of business. Mail order companies have to accept a fairly high level
of bad debts, while retailing organizations should maintain very low
levels of ratio. The actual ratio is compared with the target or norm
to decide whether it is acceptable or not.munotes.in

Page 64

646.Creditors Turnover Ratio
The measurement of the credit turnover period shows the
average time taken to pay for goods and services purchased by the
company. The formula is:
Net Credit Purchase
Average Creditors
Here purchases refer to net credit purchases and average
creditors are given by opening creditors and bills payable + closing
creditors and bills payable divided by two.
In general, the longer the credit period achieved the better,
because delays in payment mean that the operations of the
company are being financed inter est free by suppliers or materials.
But there will be a point beyond which delays in payment will
damage relationships with suppliers which, if they are operating in a
seller’s market, may harm the company. If too long a period is
taken to pay creditors, t he credit rating of the company may suffer,
thereby making it more difficult to obtain supplies in the future.
7.Creditors Payment Period or Creditors Velocity Ratio
Average Creditors365Credit Purchasesin days
Generally, payment period of 50 to 60 days is considered ideal.
8.Fixed Assets Turnover Ratio
This ratio will be analysed further with ratios for each main
category of asset. This is a difficult set of ratios to interpret as asset
values are based on historical cost. An increase in the fixed asset
figure ma y result from the replacement of an asset at an increased
price or the purchase of an additional asset intended to increase
production capacity. The formula is:
Sales
Fixed Assets
The ratio of the accumulated depreciation provision to the
total of fixed assets at cost might be used as an indicator of the
average age of the assets, particularly when depreciation rates are
noted in the accounts. The ratio of sales value per square foot of
floor space occupied is particularly significant for trading concern s,
such as a wholesale warehouse or a departmental store.
9. Total Assets Turnover Ratio
This ratio indicates the number of times total assets are
being turned over in a year. The Formula is
Sales
Total Assetsmunotes.in

Page 65

65The higher the ratio indicates overtradi ng of total assets,
while a low ratio indicates idle capacity.
10. Working Capital Turnover Ratio
This ratio is calculated as follows:
Sales
Working Capital
This ratio indicates the extent of working capital turned over
in achieving sale of the firm.
11. Sales to Capital Employed Ratio
This ratio is ascertained by dividing sales with capital
employed. The formula is:
Sales
Capital Employed
This ratio indicates efficiency in utilization of capital
employed in generating revenue.
3.9 PROFITABILIT Y RATIOS
The purpose of study and analysis of profitability ratios are to
help assessing the adequacy of profits earned by the company and
also to discover whether profitability is increasing or declining. The
profitability of the firm is the net result of a large number of policies
and decisions. The profitability ratios show the combined effects of
liquidity, asset management and debt management on operating
results. Profitability ratios are measured with reference to sales,
capital employed, total ass ets employed, share holders funds etc.
The major profitability rates are as follows:
1. Gross Profit Margin
The gross profit margin is calculated as follows:
Sales - Cost of Goods Sold Gross Profit100 100SalesSales 
The ratio measures the gross profit margin on the total net
sales made by the company. The gross profit represents the
excess of sales proceeds during the period under observation over
their cost, before taking into account administration, selling and
distribution and financing charges. The ratio measures the
efficien cy of the company’s operations and this can also be
compared with the previous year’s results to ascertain the
efficiency.munotes.in

Page 66

662. Net Profit Margin
Theratio is calculated as follows :
Net Profit before interest and Tax100Sales
The ratio is designed to focus attenti on on the net profit
margin arising from business operations before interest and tax is
deducted. The convention is to express profit after tax and interest
as a percentage of sales. A drawback is that the percentage which
results varies depending on the s ources employed to finance
business activity; interest is charged above the line while dividends
are deducted below the line. It is for this reason that net profit i.e.
earnings before interest and tax (EBIT) is used. This ratio reflects
net profit margin on the total sales after deducting all expenses but
before deducting interest and taxation. This ratio measures the
efficiency of operation of the company. The net profit is arrived at
from gross profit after deducting administration, selling and
distribut ion expenses.
3. Cash Profit Ratio
Cash profit ratio measures the cash generation in the
business as a result of the operations expressed in terms of sales.
The formula is:
Cash Profit×100SalesCash profit = Net profit + Depreciation
The cas h profit ratio is a more reliable indicator of
performance where there are sharp fluctuations in the profit before
tax and net profit from year to year owing to difference in
depreciation charged. Cash profit ratio evaluates the efficiency of
operations in terms of cash generation and is not affected by the
method of depreciation charged. It also facilitate inter firm
comparison of performance since different methods of depreciation
may be adopted by different companies.
4. Return on Total Assets
This ra tion is calculated as follows:
Net Profit after Tax×100Total Assets
The profitability of the firm is measured by establishing
relation of net profit with the total assets of the organization. This
ratio indicates the efficiency of utilization of assets in gener ating
revenue.munotes.in

Page 67

675. Return on Shareholders’ Funds or Return on Net worth
This ration expresses the net profit in terms of the equity
shareholders funds. This ratio is an important yardstick of
performance for equity shareholders since it indicates the retu rn on
the funds employed by them. However, this measure is based on
the historical net worth and will be high for old plants and low for
new plants.
Net Profit after interest and Tax100Net worth
Net worth = Equity capital + Reserves and Surplus
The factor which motivat es shareholders to invest in a
company is the expectation of an adequate rate of return on their
funds and periodically, they want to assess the rate of return
earned in order to decide whether to continue with their investment.
This ratio is useful in mea suring the rate of return as a percentage
of the book value of shareholders equity.
3.10 OPERATING RATIOS
The ratio of all operating expenses (i.e. materials used
labour, factory overheads, administration and selling expenses) to
sales is the operating ratio. A comparison of the operating ration
would indicate whether the cost content is high or low in the figure
of sales. If the annual comparison shows that the sales has
increased the management would be naturally interested and
concerned to know as to which element of the cost has gone up. It
is not necessary that the management should be concerned only
when the operating ratio goes up. If the operating ratio has fallen,
though the unit selling price has remained the same, still the
position need analy sis as it may be the sum total of efficiency in
certain departments and inefficiency in others. A dynamic
management should be interested in making a complete analysis. It
is, therefore, necessary to break -up the operating ratio into various
cost ratios. T he major components of cost are: material, labour and
overheads. Therefore, it is worthwhile to classify the cost ratio as:
Materials ConsumedMaterials Cost Ratio = ×100SalesLabour CostLabour Cost Ratio = ×100SalesFactory ExpensesFactory Overhead Ratio = ×100Salesmunotes.in

Page 68

68Administrative ExpensesAdministrative Expenses Ratio = ×100SalesSelling and Distribution ExpensesSelling and Distribution Expenses Ratio = ×100Sales
Cost of Goods Sold+Operating ExpensesOperating Ratio = ×100Net Sales 
Generally all these ratios are expressed in terms of
percentage. Then total up all the operating ratios. This is deducted
from 100 will be equal to the net profit ratio. If possible, the total
expenditure for eff ecting sales should be divided into two
categories, viz. fixed and variable and then ratios should be worked
out.
3.11 MARKET BASED RATIOS
The market based ratios relates the firm’s stock price to its
earnings and book value per share. These ratios giv em a n a g e m e n t
an indication of what investors think of the company’s past
performance and future prospects. If firm’s profitability, solvency
and turnover ratios are good, then the market based ratios will be
high and its share price is also expected to be high. The market
based ratios are as follows:
1. Earnings Per Share (EPS)
The objective of Financial management is wealth or value
maximization of a corporate entity. The value is maximized when
market price of equity shares is maximized. The use of wea lth
maximization objective or net present value maximization objective
has been advocated as an appropriate and operationally feasible
criterion to choose among the alternative financial actions. In
practice, the performance of a corporation is better judg ed in terms
of its earnings per share (EPS). The EPS is one of the important
measures of economic performance of a corporate entity. The flow
of capital to the companies under the present imperfect capital
market conditions would be made on the evaluation of EPS.
Investors lacking inside and detailed information would look upon
the EPS as the best base to take their investment decisions. A
higher EPS means better capital productivity. The ratio is calculated
as:
Net Profit after Tax -Preference Dividend
No. of Equity Shares
EPS is one of the most important ratios which measures the
net profit earned per share. EPS is one of the major factorsmunotes.in

Page 69

69affecting the dividend policy of the firm and the market prices of the
company. Growth in EPS is more relevant for pricing of shares from
absolute EPS. A steady growth in EPS year after year indicates a
good track of profitability.
2. Cash Earnings per Share
The cash earnings per share (Cash EPS) arecalculated by
dividing the net profit before depreciation with nu mber of equity
shares. The formula is:
Net Profit after Tax + Depreciation
No. of Equity Shares
This is a major reliable yardstick for measurement of
performance of companies, especially for highly capital intensive
industries where provision for depre ciation is substantial. This
measures the cash earnings per share and is also a relevant factor
for determining the price for the company’s shares. However, this
method is not as popular as EPS and is used as a supplementary
measure of performance only.
3.D i v i d e n dP a y o u tR a t i o
Dividend payout ratio is the dividend per share divided by
the earnings per share. Dividend payout indicates the extent of the
net profits distributed to the shareholders as dividend. A high
payout signifies a liberal distribution policy and a low payout reflects
conservative distribution policy. The ratio is calculated as:
Dividend per Share
Earnings per Share
4. Dividend Yield
This ratio reflects the percentage yield that an investor
receives on this investment at the current market price of the
shares. This measure is useful for investors who are interested in
yield per share rather than capital appreciation. The ratio is
calculated as:
Dividend per share×100Market Price5. Book Value
This ratio indicates the net worth per equit y share. The book
value is a reflection of the past earnings and the distribution policy
of the company. A high book value indicates that a company has
huge reserves and is a potential bonus candidate. A low book value
signifies a liberal distribution poli cy of bonus and dividends, or
alternatively, a poor track record of profitability. Book value is
considered less relevant for the market price as compared to EPS,munotes.in

Page 70

70as it reflects the past record whereas the market discounts the
future prospects. The formula is:
Equity Capital + Reserves –Profit and Loss A/c Debit balance
Total number of Equity Shares
6. Price Earnings Ratio (P/E Ratio)
The ratio indicates the market price of an equity share to the
earnings per share. It measures the number of times the earning
per share discounts the market price of any equity share. The ratio
is calculated as:
Current Market Price of Equity Share
Earnings per Shares
The ratio indicates how much an investor is prepared to pay
per rupee of earnings. The ratio helps to ascertain the value of
equity share, if the EPS and probable price -earning ratio of the
industry to which the company belongs. The intrinsic value of share
may be more or less than the market value which is influenced by
company’s track record and dividen d distribution policy, speculative
trading, state of economy, efficiency of management, capital
gearing etc. Price -earning approach to share valuation is simple
and more popular. This ratio reflects the market’s assessment of
the future earnings potential of the company. A ratio reflects high
earnings potential and a low ratio reflects the low earnings
potential. The ratio reflects the market’s confidence on company’s
equity.
7. Market Price to Book Value Ratio (P/BV ratio)
This ratio measures the relati onship between the accounting
value of the firm’s assets and the market price of its stock. The ratio
is calculated by dividing the stock price per share by the book value
of share. The ratio is calculated as:
Market Price per Share
Book Value per Share
Generally the higher the rate of return a firm is earning on its
common equity the higher will be the P/B V ratio. In case of growth
firms i.e. firms with high growth of sales and earnings will have this
ratio higher than 1, for the reason that the poten tial future growth in
earnings is reflected in the current stock price. Where as the book
value of equity is based on historical costs and it does not consider
the potential growth.munotes.in

Page 71

713.12 QUESTIONS
(I) Theory Questions:
1. What is window dressing of Cu rrent Ration and Trading on
Equity?
2. “Ration Analysis is only a tool and not a final decision.”
Discuss.
3. Ratio analysis is only a technique for making judgments and
nota substitute for it. Comment
4. What are the accounting ratios?
5. Explain the importance of Ratio Analysis.
6. What is ratio analysis? What are its limitations?
7. What is the ratio analysis?
8. What are the objects of ratio analysis?
9. State the different ratios according to Conventional and
Functional Classification.
10. State the signif icance of the following ratios :
a)Earnings per share (b) Pricing Earning Ratio (c) Return on
Capital Employed (including Long -term Borrowings) (d)
Creditors Turnover Ratio.
11. What is a ‘ratio’? What are the limitations of ratio analysis?
12. What is vertical analysis?
13. State the different modes of expressing ratios.
14. Distinguish between balance sheet ratios and revenue
statement ratios.
15. How would you test the short -term solvency of a company?
16. What is the purpose of a li quid ratio?
17. What is a proprietary ratio? State its significance.
18. Explain the concept of over -capitalisation and
undercapitalisation.
19. What is over -trading and under -trading?
20. Explain the following ratios help the management in
interpretati on of financial data
a. Acid Test ratio.
b. Stock -working capital ratio.
c. Proprietary ratio.
(II) State whether the following statement are TRUE or FALSE
1) The standard for current ratio is 2.
2) Difference in the methods of valuation of inventory between
firms is one of the difficulties of Ratio Analysis.munotes.in

Page 72

723) The ratioEarnings before Interest and TaxSalesmeasures stability of
the firm.
4) Ratio Analysis enables inter -firm comparison.
5) Inventory turnover ratio isCost of Goods soldTotal Inventory6) Owed fu nds are internal source of finance.
7) Advance to suppliers isclassified as Quick assets in vertical
statements.
8) High Stock turnover ratio indicates high cost of Goods sold.
9) Unclaimed dividends are classified as current liabilities in
vertical fi nancial statement.
10) A Liquid ratio of 0.5:1 indicates over investment.
11) The firm short term solvency is indicated by its current ratio.
12) When the firm does not have preference share capital it’s
Capital Gearing Ratio and Debt Equity Ration valu es are the
same.
13) Debt -Equity Ratio can be calculated in two ways :
a)DebtEquityb)DebtDebt Equity
14) Higher stock turnover ratio means loss due to obsolete stock
would be maximum.
III. Short Notes
1. Functional Cla ssification of Ratios.
2. Retained Earnings.
3. Explain the significance of capital gearing ratio.
4. What are liquidity ratios and what is their significance?
5. Profitability Ratios.
6. Accounting Ratios measuring Profitability and Solvency.
7. Profitability ratios vis -à-vis investments.
8. Current Ratio.
9. Capital Gearing Ratio.
10. Ratio analysis
11. Trading on equity
12. Debtor’s Velocity
13. Classification of Accounting Ratios.munotes.in

Page 73

7314. Debtors Turnover Ratio.
15. Debtor Turnover Ratio and C reditor Turnover Ratio.
16. Importance of balance sheet Ratios.
17. Return on Capital Employed
18. Short term and Long term Ratios.
19. Limitations of Ratio Analysis.
20. Trading on Equity
IV. Multiple Choice Question s:
1. A very high current ratio will:
a)increase the profitability
c)not affect the profitability
b)adverse impact on profitability
d)none of the above.
2. One of the following is not an absolute liquid asset :
a)cash in hand c)bills receivable
b)cash at bank d)marketable investments
3. Fixed interest bearing funds do not include one of the following
a)debentures c)preference share capital
b)long-term investments d)public deposits
4.The ratio of sales value per square foot of floor space is not
suitab le for :
a)Trading concerns c)wholesale warehouse
b)Departmental stores d)manufacturing concern
5.The term cash profit indicates :
a)Gross profit + Interest + Depreciation
b)Net profit + Depreciation
c)Net profit –Interest –Tax
d)Net profit before depreciation, interest and tax.
6.The return on equity indicates:
a)Measure of profitability.
b)The efficiency in use of assets in achieving sales.
c)Measure of leverage
d)All of the above,
7.Return on investment can be in creased by :
a)Increasing the profit margin
c)reduction of invested capital
b)Increasing the investment turnover
d)all of the above.munotes.in

Page 74

748.Profit margin (net) of B.S. Ltd. is 7% while turnover is 3 times of
its capital. The return on investment of the concern is ……….
a)20% c)21%
b)18% d)19%
9.If current ratio is given as 2.5, liquid assets are Rs. 60,000, then
the value of the stock will be
a)Rs. 60,000 c)Rs. 20,000
b)Rs. 40,000 d)Rs. 30,000
10.If the current ratio and liquid ration of a firm are 2.2 and o.8
respectively and its Current liabilities is Rs. 10 lakhs. The value
of stock held by the firm is Rs. ___ lakhs.
a)12 c)16
b)14 d)none of the above
11.The current ratio BM Ltd, is 2:1, while quick ratio is 1.80:1. If the
current liabilities are Rs. 40,000, the value of stock will be :
a)Rs. 6,400 c)Rs. 10,000
b)Rs. 8,000 d)Rs. 12,000
12.Warfied company having net working capital of Rs. 3 lakh has
the current ratio of 1:8 and liquid ratio of 1:6. Its value of stock
is:
a)Rs. 55,000 c)Rs. 75,000
b)Rs. 65,000 d)Rs. 85,000
13.Asset management ratios are used to measure the speed with
various accounts are converted into :
a)Fixed assets or current assets
b)Sales or inventory
c)sales or cash
d)cash or bank
14.A firm seeks to increase its current ration from 1:5 before its
closing date of the accounts. The action that would make it
possible is :
a)Delaying payment of salaries
b)Increase charge for depreciation.
c)Making cash payment to creditors.
d)Selling marketable securities for cash at book value.

munotes.in

Page 75

75Unit-4
RATIO ANALYS IS
PART II
Unit Structure :
4.1 Objectives
4.2 Ratios Summarized
4.3 Solved Illustrations
4.4 Exercise
4.1 OBJECTIVES
After studying the unit the students will be able to solve the
problems.
4.2 RATIOS SUMMARIZED
munotes.in

Page 76

76
munotes.in

Page 77

77
munotes.in

Page 78

784.3 SOLVED ILLUSTRATIONS
Problem 1
The following figures relate to the trading activities of Hind Traders
for the year ended 30thJune, 19X1:
You are required to (1) rearrange the above figures in a form
suitable for analysis, and (2) show separately the following ratios :
(i) gross profit ratio; (ii) operating ratio; (iii) stock turnover ratio.
(C.A. adapted)
Solution
HIND TRADERS LTD.
Profit and Loss Statement
munotes.in

Page 79

79
Gross profit ratio:Rs. 6,00,000=0.40 or 40%Rs. 15,00,000
Operating ratio =Cost of Goods Sol d + Operating Expenses
Sales
=11,40,000
15,00,000
= 0.76 or 76%
Stock turnover ratioCost of goods soldAverage stockRs. 9,00,000=3.43Rs. 2,62,125times
Problem 2
Towards the end of 19 X 1 the directo rs of wholesale Merchants
Ltd., decided to expand their business. The annual accounts of the
company for 19 X 1 may be summarized as follows
WHOLESALE MERCHANTS LTD
Financial statements
(Rs)
munotes.in

Page 80

80
You are informed that (a) All sales were from stocks in the
company’s ware house, (b) The range of merchandise was not
changed buying prices remained steady throughout the two years
(c) Budgeted total sales for 19X2 were Rs. 3,90,000 (d) The
debenture loan was received on 1stJanuary 19X2, and additional
fixed assets were purchased on that date.munotes.in

Page 81

81You are required to state the internal accounting ratios that
you would use in this type of business to assist the management of
the company in measuring the efficiency of its operation, including
its use of capital .
Your answer should name the ratios and give the figures
(calculated to one decimal place) for 19X1 and 19X2, together with
possible reasons for changes in the ratios for the two years. Ratios
relating to capital employed should be based on the capital at the
end. Ignore taxation.
(C.A. adapted)
Solution
The following ratios are calcula ted for Wholesale Merchants
Ltd:
RATIOS FOR WHOLE SALE MERCHANT LTD.
munotes.in

Page 82

82Note :EBIT fo r 19X1 and 19X2 respectively is : Rs. 21,000 and Rs.
23,800 + 2,800 = Rs. 26,6 00.
Comments :The return on capital employed has fallen from 15% in
19X1 to 12.9in 19X2. The reason lies in the sales to capital ratio
which has also fallen in 19X2. The increase in capital employed has
not been profitably utilized. The increased capital seems to have
been blocked in stock and debtors.
It will be noticed that the gross margin ratio decreased from
21.3% in 19X1 to 20.3% in 19X2. This may be attributed to reduced
selling price or granting of trade discounts to bulk orders. The
operating r atio (expense to sales ratio) has fallen in 19X1 by 1%
and this had a slight impact on net profit ratio which has increased
by 0.1%.
The short term solvency of the company, reflected by current
ratio and quick ratio, is more or less constant. However, t here has
been deteriorating in the stock turnover and debtors turnover ratios.
This implies the company is holding stocks for longer periods and
allowing longer credit periods to customers.
There is no threat to the long -term solvency of the company.
Itdid not use any long -term debt in 19X1. A debenture loan of Rs.
42,000 is taken in 19X2 and is about 0.26 of the equity funds. By a
normal criterion, the company could have a debt equity ratio of 2:1.
Problem 3
On the basis of financial statements of ABC Ltd, given
below, comment on the asset utilization and profitability of ABC Ltd.
Income Statement ABC Ltd. (Rs. in crore)
munotes.in

Page 83

83
Balance Sheet ABC Ltd.
(Rs. in crore)
munotes.in

Page 84

84
Solution
munotes.in

Page 85

85Ideally inventory turnover ratio is calculated as Cost of
Goods Sold/Average Inventory. However, in the absence of
information related to cost of goods sold, we make use of sales
figure. Similarly, average inventory is the average of opening and
closing inventory. However, in the absence of information we may
use closing inventory.
Comments
Fixed assets turnover ration indicates the efficient utilization of
fixed assets. It indicates to what extent fixed assets are
contributing in the generation of sales. In case of ABC Ltd, fixed
assets turnover ratio is decl ining. This indicates inefficient
management of fixed assets.
Inventory turnover ratio indicates how quickly inventory is
converted into sales. The higher the inventory turnover ratio, the
better it is for the organization. In case of ABC Ltd., inventory
turnover ratio is more or less constant and is good enough. The
corresponding inventory holding period is around 1.5 months,
which is reasonably less. This indicates that inventory is not
moving very slowly resulting into losses.
Debtors turnover ratio in dicates how quickly outstanding debts
are collected to generate cash. A high debtor’s turnover ratio is
a sign of efficient collection department. In case of ABC Ltd.
Debtors’ turnover ratio increased in the year 2003 but declined
again in 2004 depicting i nefficiency of the debt collection
department.
Net profit margin has increased continuously which is a healthy
sign. This increase in profit margin is probably due to better cost
management. However, this can be confirmed only in the light of
further info rmation.
Return on net worth depicts a fluctuating trend but it is moving
within a narrow range. However, RONW is also reasonably
high.
Problem 4
From the following details, furnished by Globe Traders for
the year ended on 31.3.2009, prepare the Balanc es h e e ta so nt h a t
date :
munotes.in

Page 86

86The firm sells its products only on credit. Credit sales for the
year ended 31 -3-2009 amounted to Rs. 120 lakhs.
Solution
Working Notes
1Calculation of Cost of Sales
Sales = Rs. 120 lakhs (given)
Gross profit ratio =25% (given)
Cost of sales =7 5 %
Cost of sales = Rs. 120 lakhs x 75/100 Rs. 90 lakhs
2Calculation of Debtors
Average collection period =Debtors12 months 1½ monthsCredit sales 
1.5 months =Debtors12 monthsRs.120 lakhs
Debtors x 12 months = Rs.120 lakhs x 1.5 months
Debtors =1 8 0 / 1 2R s . 15 lakhs
3Calculation of Fixed AssetsCost of salesFixed assets=1 . 2( g i v e n )
= Rs. 90 lakhs
Fixed assets =1 . 2
1.2 x Fixed assets = Rs. 90 lakhs
Fixed assets = Rs. 90 lakhs / 1.2 Rs. 75 lakhs
4Calculation of Closing stockCost of salesClosing stock=9( g i v e n )Rs. 90 lakhsClosing stock=9
9 x closing stock = Rs. 90 lakhs
Closing stock = Rs. 90 lakhs / 9 Rs. 10 lakhs
5Calculation of Current Assets
and Current Liabilities
Current ratio =1 . 7 5( g i v e n )Current assets CACurrent liabilities C.L.=1 . 7 5
CA =1 . 7 5C LCA-StockCL=1 . 2 5CA-10CL=1 . 2 5
CA =1 . 2 5C L+1 0
1.75 CL =1 . 2 5C L+1 0
0.50 CL =1 0
CL =1 0 / 0 . 5 0=2 0munotes.in

Page 87

87Current liabilities =Rs. 20 lakhs
Current assets = Rs. 20 lakhs x .125) + Rs. 10 lakhs
Rs. 35 lakhs
6Calculation of Net worthRs. 75 lakhsNet worth=1 . 2 5
Net worth = Rs. 60 lakhs
7Calculation of Equity share
capitalReserveCapital=0 . 2( g i v e n )
Fixed assets1.25Capital + Reserve
751.25

Capital + 0.2 capital
1.25 x 1.2 capital = 75
1.5 capital = 75
Capital = 75/1.5 = 50
Capital = Rs. 50 lakhs
8Calculation of DebtDebtEquity=0 . 6( g i v e n )Debt50=0 . 6
Debt =0 . 6x5 0 R s .3 0l a k h s
9Calculation of Reserves
Reserves =0 . 2E q u i t y=0 . 2x5 0 R s .1 0l a k h s
Globe Traders
Balance Sheet as 31/3/2009
munotes.in

Page 88

88Problem 5
With the following ratios and other informatio n, prepare Trading
Account, Profit and Loss Account and Balance sheet of Anand :
Solution
Working notes
munotes.in

Page 89

89
Trading and Profit and Loss Account of Sri Anand for the ended.
Balance Sheet of Shri Anand as at ………
munotes.in

Page 90

90Illustration 6
The Tradin g and Profit and Loss Account of SKSS Ltd., for the year
ended 31stMarch, 2007 isgiven below :
You are required to calculate operating ratios.
Solution :
Calculation of Operating Ratios :
1.Materials Cost Ratio
Materials consumed100 24,00,000Sales
11,50,00047.92%100

Note : Materials consumed
=Opening stock + Purchases –Closing stock
=2,50,000 + 10,50,000 –1,50,000 = Rs. 11,50,000
2. Labour Cost Ratio
Labour cost 4,00,000×100 = 100 16.6%Sales 24,00,000munotes.in

Page 91

913.Factory Expenses Ratio
Factory expenses 2,00,000×100 = 100 8.33%Sales 24,00,000
4.Administrative Expenses R atio
Administrative expenses 2,30,000×100 = 100 9.58%Sales 24,00,000
5.Selling and Distribution Expenses Ratio
Selling and distribution expenses 1,00,000×100 = 100 4.17%Sales 24,00,000 
6.Operating Ratio
Cost of goods sold + Operating expenses×100Sales
17,50,000+3,30,000= 100 86.67%24,00,000 
Working Note
i)Cost of Goods Sold
=opening stock + Purchase + Wages + Factory expenses –
Closing stock
=2,50,000 + 10,50,000 + 4,00,000 + 2,00,000 –1,50,000
=Rs. 17,50,000
ii)Operating Expenses
=Administration expenses + Selling and distribution expenses
=2,30,000 + 1,00,000 = 3,30,000
Net Profit Ratio
=100% -Operating Ratio =1 0 0 % -86.67% = 13.33%
Illustration 7
From the financial information of Yahoo Ltd., given below,
calculate activity or turnover ratios :
Balance sheet as on 31stMarch, 2007
munotes.in

Page 92

92
Tax @ 50% loan installment payable during the year Rs. 3, 00,000.
Equity dividend declared during the year @ 18% you are required
to calculate long -term solvency ratios.
Solution
1.Debt -Equity Ratio
Long-term debt 18,00,000= =0.33%Shareholders funds 30,00,000+10,00,000+14 ,00,000
The company’s long -term solvency is more satisfactory. The
debt-equity ratio of the compan yi s0 : 3 3 : 1a n di ti sw e l lw i t hi nt h e
accepted norm of 2:1. Since the proportion of debt to equity is low,
the company is said to be low geared and could not reap the
benefit of trading on equity.
2.Shareholders Equity Ratio
Shareholders equity 54,00,000= =0.6Total assets (tangible) 90,00,000munotes.in

Page 93

93Since the shareholders equity is larger in proportion of 50%
to total assets. Therefore, the financial of position of the company is
stronger.
3.Long -term Debt to Shareholders Networth Ratio
Long-term debt 18,00,000= =0.33%Shareholders funds 30,00,000+10,00,000+14 ,00,000
Since the long -term debt r epresents only 33% of the
shareholders net worth leaving balance 67% to other current
liabilities, gives an indication of stronger short -term as well as long -
term solvency of the company.
4.Capital Gearing Ratio
Fixed interest bearing funds 10,00,000+18,00 ,000= =0.64%Equity Shareholders funds 30,00,000+14,00, 000
The fixed interest bearing funds represents 64% of the
equity shareholders funds. The financial risk of the company is
lower and the earning available to equity shareholders is less
vulnerable.
5.Fixed Assets to Long -term Funds Ratio
Fixed Assets 62,00,000= =0.86Long Term Funds 30,00,000+10,00,000+14,00,0 00-18,00,000
It indicates the long -term solvent position of the company.
The high ratio indicates the high proportion of long -term funds
deployed in fixed assets.
6.Proprietary Ratio
Shareholders networth 54,00,000= =0.60Total Assets 90,00,000
The high proprietary ration indicates the strong financial
position of the business.
7.Interest Cover
25,00,000=11.572,16,000time interest
An interest cover 2:1 is ideal. The high ratio of the company
indicates the low proportion of debt and the company is following a
very conservative policy in using the debt component in the capital
structure.munotes.in

Page 94

948.Debt Service Coverage Ratio
Profit after tax+interest+Depreciation
Interest + Periodic loan instalment
8,90,000+2,16,000+8,00,000 19,06,400= 3.69 times2,16,000+3,00,000 5,16,000

The higher debt service coverage ration of the company
indicates the better servicing ability of the company.
9.Preference Dividend Cover
Profit available to preference shareholders 8 ,90,400= =8.90Preference dividend 1,00,000  times
The higher preference dividend cover indicates greater
assurance to preference shareholders in getting their assure return
of 10%.
10.Equity Dividend Cover
Profit after tax-Preference dividend Profit available to Equity shareholdersEquity Dividend Equity Dividend
8,90,000-1,00,000 7,90,400= 1.465,40,000 5,40,000OR    

Illustration 8
From the following details, prepare statement of proprietary
funds with as many details as possible.
i)Stock velocity : 6
ii)Capital turnover ratio (on cost of sales) : 2
iii)Fixed assets turnover ratio (on cost of sales) :4
iv)Gross profit turnover ratio : 20 percent.
v)Debtors’ velocity : 2 months
vi)Creditors velocity : 73 days
The gross profit was Rs. 60,000. Reserves and Surplus
amount Rs. 20,000. Closing stock was Rs, 5,000 in excess of
opening stock.
Solution
1.Sales :
Gross ProfitGross Profit ratio = 100Sales
If Gross profit is Rs. 20, Sales = Rs. 100
If Gross profit is Rs. 60,000, Sales = 60,000 x 100/20
= Rs. 3,00,000
2.StockCost of goods soldStock velocity= =6Average stockmunotes.in

Page 95

95Cost of goods sold = Sales –Gross profit
= Rs. 3,00,000 –Rs. 60,000
= Rs. 2,4 0,000
2,40,000== 6Average stock
6 x Average stock = 2,40,000
Average stock = 2,40,000 + 6 = Rs. 40,000
Opening stock+Closing stockAverage stock= =Rs. 40,0002Total of stocks (40,000 x 2) = Rs. 80,000
Rs. 5,000Less: Excess=Rs. 75,000
75,000Opening stock= =Rs.37,5002Closing stock = 37,500 + 5,000 = Rs. 42,500
3.Debtors :
Debtors + Bills receivable× No. of workingdays = 2 months  Debtors velocity = Credit sales
There are no bills receivables , Hence,
DebtorsDebtors velocity ×12=23,00,000
Adopting cross multiplication,
3,00,000 2= =Rs.50,00012Debtors4.Creditors :
Purchase = Cost of goods sold + Cl osing stock –Opening stock
= Rs. 2,40,000 + Rs. 42,500 –Rs. 37,500
= Rs. 2,45,000
There is no bills payable, Hence,
Creditors’ velocityCreditors velocity = 265=732,45,000
73 2,45,000Creditors = =Rs.49,000365Creditors 

5.Fixed assets :
Fixed assets turnover ratio (based on cost of sales)Cost of sales== 4Fixed assets2,40,000== 4Fixed assets4 x Fixed assets = Rs. 2,40,000
Fixed assets2,40,000= =Rs.60,0004munotes.in

Page 96

966.Share Capital :
Cost of sales== 2Total capital (or) Proprietary fund2,40,000== 2Proprietary fund 

2 x Proprietary fund = Rs. 2,40,000
2,40,000Proprietary fund = =Rs.1,20,0002
Proprietary fund = Rs. 1,20,000Rs.20,000Less: Reserves and Surplus =Rs.1,00,0007.Cash
Balance sheet
Statement of Proprietary Funds
munotes.in

Page 97

974.4 EXERCISE
PRACTICLES PROBLEMS
1.A company has Rs. 1, 00,000 in inventory and Rs. 40,000 cost
of goods sold. Management has set a 4:1 goal for inventory
turnover. How much is the inventory over or under the amount
that would give this 4:1 ratio.
2.A company has current liabilities of Rs. 2, 00,000 a mortgage of
Rs. 3,00,000 and bonds of Rs. 5,00,000. Its total equity is Rs.
15,00,000. What is i ts debt -equity ratio?
3.A company has a net income after tax of Rs. 4,00,000 and pays
cash dividends of Rs. 2,40,000 on its 2,00,000 shares of
outstanding stock at a time when the stock is selling for Rs. 20.
What is the dividend yield and dividend pay o ut of the
company?
4.A company has a net income after tax of Rs. 2,00,000 and
80,000 share outstanding, selling at a market price of Rs. 30.
What is the company’s P/E ratio?
5.If a company has sales of Rs. 2,00,000 and average accounts
receivable at Rs . 40,000, what is the accounts receivable
turnover and average collection period?
6.Following information are available from recent accounts of M.
Ltd.
It is proposed to enter an entirely new market with a product
which has not been handled before. This will lead to an additional
sales of Rs. 2,00,000 having a gross profit rate of 20%. Customers
will except 60 days a credit and additional stock of raw materials
equal to three months’ usage will be needed. Raw material costs,
on existing products as with the new product, account for 75% of
cost of sales.
If the proposal is implemented, how it affect company’s
ratios (stock turnover ratio and debt collection period)?
7.You are required to make a quick financial projection (i.e.
Projected Income St atement and Projected Balance sheet) for the
year 2010 -11 on the basis of the following limited information :munotes.in

Page 98

982009 -10
Balance Sheet as on 31/3/2010
What will be the dividend rate on the basis of above dividend
payout ratio? You may make necessary assumption.
8.Given below are cash position ratios of MRD Ltd., and
Industry Average. Industry
Average is arrived at by taking position of 25 companies of
the similar trade.
How do you feel about the cash position of MRD Ltd?
Comparative Balan ce Sheet for the year ended 31stMarch 2008 is
as follows :
munotes.in

Page 99

99
Comparative Profit and Loss A/c for the year ended 31stMarch,
2006 is as follows :
Calculate the following Ratios and Comment:
a)Current ratio e)Proprietary ratio
b)Liquid ratio f)Debt equity ratio
c)Gross profit ratio g)Debtors turnover ratio
d)Operating ratio h)Selling expense ratio
(M.U. B.Com, October 2000, Adapted)
9. From the following Balance sheet of Bapat Ltd., as on
31/12/2005 and the Trading and Profit and Los sA / cf o rt h e year
ended 31 -12-2005, calculate the following ratios.
a)Current Ratio b)Liquid Ratio
c)Stock Working Capital Ratio d)Operating Ratio
e)Stock Turnover Ratio f)Debtors Collection Period
g)Return of Equity Capital h)Gross P rofit Ratio
i)Proprietary Ratio j)Expense Ratio.munotes.in

Page 100

100
Trading and Profit and Loss Account
(M.U., B.Com April 2002, Adapted)
10.From the following information, prepare a balance sheet in
vertical form and calculate :
a)Capital Gearing Ratio d)Liquid Ratio
b)Proprietary Ratio e)Debt -Equity Ratio
c)Current Ratio f)Stock to working capital Ratiomunotes.in

Page 101

101
(M.U. B.Com., October 2002)
munotes.in

Page 102

102Unit-5
COST OF CAPITAL
Unit Structure:
5.0 Objectives
5.1 Introduction
5.2 Definition o f Cost of Capital
5.3 Measurement of Cost of Capital
5.4 Cost of Debt
5.5 Cost of Bonds
5.6 Cost of Preference Shares
5.7 Cost of Equity
5.8 Cost of Retained Earnings
5.9 Weighted Average Cost of Capital (WACC)
5.10 Exercises
5.0 OBJECTIVES
After studying this chapter you will be able to:
Understand the concept of Cost of Capital
Understand the different sources of capital
Understand the cost of employing e ach of these sources of
capital
Know the concept of w eighted average cost of capital
The imp ortance of cost of capital in financial management
5.1 INTRODUCTION
The financing decision relates to the composition of relative
proportion of various sources of finance. The sources are;
1.Owned Capital -i.e. Equity Share Capital, Preference Share
Capital, Accumulated profits.
2.Borrowed Capital -: Debentures, loans from financial institutions .
The financial management weighs the merits and demerits
of different sources of finance while taking the financing decisions.
Whether the companies choose shareholders funds or a
combination of both, each type of fund carries a cost.munotes.in

Page 103

103The cost of equity is the minimum return the shareholders
would have received if they had invested elsewhere. Borrowed fund
cost involves interest payment.
Both types of fund incur cost and this is the cost of capital to
the company. This means, cost of capital is the minimum return
expected by the company.
Whenever funds are to be raised to finance investments,
capital structure decision is involved.
A demand for raising fun ds generates a new capital structure
since a decision has to be made as to the quantity and forms of
financing.
5.2 MEANING OF COST OF CAPITAL
In simple terms cost of capital refers to the discount rate that
is used in determining the present value of th e estimated future
cash flows of the business/new project and eventually deciding
whether the business/new project is worth undertaking or not.
It is also the minimum rate of return that a firm must earn on
its investment which will maintain the market v alue of shares at its
current level.
It can also be stated as the opportunity cost of an
investment, i.e. the rate of return that company would otherwise be
able to earn at the same risk level as the investment that has been
selected. For example, when an investor purchases a stock in a
company, he/she expects to see a return on that investment. Since
the individual expects to get back more than his/her initial
investment, the cost of capital is equal to this minimum return that
the investor expects to rec eive which is termed as investor’s
opportunity cost.
The cost of each source of capital is called specific cost of
capital. When these specific costs are combined for all the sources
of capital for a business, it is termed as overall cost of capital for a
business.
5.3 MEASUREMENT OF COST OF CAPITAL
The first step in the measurement of cost of capital of the
firm is the calculation of the cost of individual sources of raising
funds. From the viewpoint of capital budgeting decisions, the long
term sources of funds are relevant as they constitute the majormunotes.in

Page 104

104sources of financing the fixed assets. In calculating the cost of
capital, therefore, the focus on long -term funds which are: -
i.Long term debt (including debentures)
ii.Preference shares
iii.Equit yC a p i t a l
iv.Retained Earnings
5.4 COST OF DEBT
The calculation of the cost of debt is relatively easy. A debt
may be in the form of Bond or Debenture. A Bond is a long term
debt instrument or security. Bonds are issued by the government.
Therefore, th ey do not have any risk of default. The government
honours obligations on its Bonds. Bonds of the public sector
companies in India are generally secured, but they are not free
from the risk of default.
The private sector companies also issue bonds, which are
called as Debentures in India. A company in India can issue
secured or unsecured debentures.
5.4.1 COST OF DEBENTURES
The cost of debentures and long term loans is the
contractual interest rate adjusted further for the tax liability of the
company. Fo r a company, the higher the interest charges, the lower
the amount of tax payable by the company. The interest on
debentures or bonds is debited to the profit and loss account.
Therefore, the taxable profit of the company is reduced. It is an
indirect savi ng to the company. Therefore the cost of debt capital is
reduced to the extent of the tax liability.
Illustration 1: Two companies X and Y are having their capital
structure as follows;
Company X Company Y
Earnings before interest and taxes (EBIT) 100 100
(Rs. In lakhs)
Interest (I) (12% ) - 40
Profit before tax (PBT) 100 60
Tax (T) @3 5 % 35 21
Profit after Tax (PAT) 65 39
The tax rate applicable to the company is 35 percent.
Solution :
Cost of Debt = (I -t) where I = interest rate an dt=t a xr a t e
Cost of debt of X = 0, there is no debt.
Cost of debt of Y = (I -T) = (12 -35%) =12 -4.2 = 7.8%munotes.in

Page 105

105The important point to remember, while calculating the
average cost of capital, is that the post -tax cost of debt should be
used and not the pr e-tax cost of debt.
5.4.2 COST OF IRREDEEMABLE DEBENTURES
Cost of debentures not redeemable during the life time of the
company.
where,
Kd=cost of debt after tax
I=Annual interest payment
NP=Net proceeds of debentures
t=Tax rate
Illust ration 2
A company issues 1,000 15% debentures of the face value
of`100 each at a discount of `5. The under -writing and other
costs are `5000 / -for the total issue. . The interest per annum is `
15,000. The income tax rate is 40%. Calculate the cost of Debt.
Solution
The net proceeds of the debenture = 1000 x 95 = `95,000
`95,000-`5,000 = `90,000
Net proceeds per debenture = 90 ,000/1 ,000 = `90
Though the interest on debenture is 15%, the net cost of debenture
is 10%.
5.4.3 Cos t of Redeemable debentures
If the debentures are redeemable after the expiry of a fixed
period, the cost of debentures would be:munotes.in

Page 106

106
where,
I =A n n u a li n t e r e s tp a y m e n t
NP = Net proceeds of debentures
RV =R e d e m p t i o nv a l u e so fd e b e n t u r e s
t =T a xr a t e
n =L i f eo fd e b e n t u r e s
Illustration 3:
A company issued 10,000, 10% debentures of Rs 100 each
on 1.4.2007 to be matured on 1.4.2012. If the market price of the
debenture is Rs.80. Compute the cost of debt assuming 35% tax
rate.
Solution :
Illustration 4
Five years ago, Sonata Limited issued 12 per cent
irredeemable debentures at Rs. 103, at Rs. 3 premium to their par
value of Rs 100. The current market price of these debentures is
Rs. 94. If the company pays corporate tax at a rate of 35 per cent
what is its current cost of debenture capital?
Solution:
Kd= 12/94 = 12.8 per cent
Kd(after tax) = 12.8 x (1 -0.35) = 8.3 per centmunotes.in

Page 107

1075.5 COST OF BONDS
It is easy to find out the present value of a bond since its
cash flows and the discount rate can be determined easily. If there
is no risk of default, then there is no difficulty in calculating the cash
flows associated with a bond. The expected cash flows consist of
annual interest payments plus repayment of principal. The
appropriate capitalization or discount rate would depend upon the
risk of bond. The risk in holding the government bond is less than
the risk associated with a debenture issued by a company.
Therefore, a lower discount rate would be applied to the cash flows
of the government bond a nd a higher rate to the cash flows of the
company debenture.
5.6 COST OF PREFERENCE SHARES
The cost of preference share capital is the dividend expected
by its holders. Though payment of dividend is not mandatory, non -
payment may result in exercising of voting rights by them.
The payment of preference dividend is not adjusted for taxes
as they are paid after taxes and is not deductible.
The cost of preference share capital is calculated by dividing
the fixed dividend per share by the price per preferenc e share.
Illustration 5:
Suzlon Energy has issued preference shares at Rs. 100 per share,
with a stated dividend of Rs. 12% and a flotation cost of 3% what is
the cost of preference share?
Solution
 
pPreference DividendK=Market Price of Preference Share 1-flotation costRs.12= 12 / 97 12.37%Rs.100 1-0.03 
5.6.1 COST OF IRREDEEMAB LE PREFERENCE SHARES
Cost of irredeemable preference sharesPDPO
Where,
PD = Annual preference dividend
PO = Net proceeds in issue of preference shares.munotes.in

Page 108

108Cost of irredeemable preference shares where Dividend Tax
is paid over the act ual dividend payment1PD DtPO
Where,
PD = Annual preference dividend
PO = Net proceeds in issue of preference shares
Dt = Tax on preference dividend
Illustration 6:
X Ltd. issued 2,000 10% preference shares of Rs 100 each
at Rs. 95 eac h. Calculate the cost of preference shares.
Solution
5.6.2 COST OF REDEEMABLE PREFERENCE SHARES:
If the preference shares are redeemable after the expiry of a
fixed period the cost of preference shares would be:/2pPD RV NP NKRV NP
Wher e,
PD = Annual Preference Dividend
RV = Redemption value of Preference Shares
NP = Net proceeds on issue of Preference Shares
N = Life of Preference Shares
However, since dividend of preference shares is not allowed
as deduction from income for e tax purposes, there is no question
of tax advantage in the case of cost of preference shares.
The cost of redeemable preference share could also be
calculated as the discount rate that equates the net proceeds of the
sale of preference shares with the presen tv a l u eo ft h ef u t u r e
dividends and principal payments.
Thus, in the case of debt as well as preference shares, cost
of capital is calculated by reference to the obligations incurred and
proceeds received.munotes.in

Page 109

109Illustration 7:
Y Ltd. issued 2,000 10% preferen ce shares of Rs 100 each
at Rs 95 each. The company proposes to redeem the preference
shares at the end of 10 years. Calculate the cost of preference
shares.
Solution :/2pPD RV NP NKRV NP
10 100 9510
100 95
2
0.10710.7%pK
approx      

5.7 COST OF EQUITY
It may prima facie appear that equity capital does not carry
any cost. But this is not true. The market share price is a function of
return that equity shareholders expect and get. If the company does
not meet their requirements, it will have an adverse effect o nt h e
market share price. Also, it is relatively the highest cost of capital.
Since expectations of equity holders are high, higher cost is
associated with it.
In simple words cost of equity capital is the rate of return
which equates the present value of expected dividends with the
market share price. In theory the management strives to maximize
the position of equity holders and the effort involves many
decisions.
Different methods are employed to compute the cost of
equity capital.
a)DIVIDEND PRICE APPROACH
Here, cost of equity capital is computed by dividing the
current dividend by average market price per share. However, this
method cannot be used to calculate cost of equity of units suffering
losses.
This dividend price ratio expresses the cost of equity capital
in relation to what yield the company should pay to attract
investors.munotes.in

Page 110

11010eDKP
Where,
Ke= Cost of equity
D1=A n n u a ld i v i d e n d
P0= Market price of equity
This model assumes that dividends are paid at a constant
rateto perpetuity. It ignores taxation.
Earnings and dividends do not remain constant and the price
of equity shares is also directly influenced by the growth rate in
dividends. Where earnings, dividends and equity share price all
grow at the same rate, the c ost of equity capital may be computed
as follows:
Ke=( D 1/P0)+G
Where,
D1=[ D 0(1+G)] i.e. next expected dividend
P0= Current Market price per share
G= Constant Growth Rate of dividend
Cost of newly issued shares, Kn , is estimated with the
const ant dividend growth model so as to allow for flotation costs.
Kn=(D 1/P0-F) + G
Where,
F = Amount of flotation cost per share
Illustration 8:
A company has paid a dividend of Rs 1 per share (of face
value of Rs 10 each) last year and it is expected to gr ow @10%
next year. Calculate the cost of equity if the market price of share is
Rs 50.
Solution
 1 1 0.100.10500.12 12%eDGKPor
 Ke= D + Gmunotes.in

Page 111

111b)EARNING/PRICE APPROACH:
This approach co -relates the earnings of the company with
the market price of its share.
The cost of ordinary share capital would be based upon the
expected rate of earnings of a company. The argument is that each
investor expects a certain amount of earnings, whether distributed
or not from the company in whose shares he invests.
If an invest or expects that the company in which he is going
to subscribe for shares should have at least a 20% rate of earning
the cost of ordinary share capital can be construed on this basis
Suppose the company is expected to earn 30%, the investor will be
prepared to pay Rs 150 Rs30 x 100 for each share of Rs 100.
So, cost of equity will be given by:/20eEPK   Ke= (E/P)
where,
E= Current earnings per share
P= Market share price
Since earnings do not remain constant and the price of
equity shar es is also directly influenced by the growth rate in
earning, we need to modify the above calculations with an element
of growth.
So, cost of equity will be given by:
Ke=( E / P )+G
where,
E = Current earnings per share
P = Market share price
G=A n n u a lg rowth rate of earnings
The calculation of ‘G’ (the growth rate) is an important factor
in calculating cost of equity capital. The past trend in earnings and
dividends may be used as an approximation to predict the future
growth rate if the growth rate of dividend is fairly stable in the past.
G=1 . 0( 1 + G )nwhere n is the number of years
The Earning Price Approach is similar to the dividend piece
approach; only it seeks to nullify the effect of changes in the
dividend policy.munotes.in

Page 112

112c)REALIZED YIELD APPROACH :
According to this approach, the average rate of return
realized in the past few years is historically regarded as ‘expected
return’ in the future. The yield of equity for the year is:11ttt
tDPYP
where,
Yt=Y i e l df o rt h ey e a rt
Dt=D ividend for share for end of the year t
Pt= Price per share at the end of the year t
Pt-1= Price per share at the beginning and at the end of the year t
This approach provides a single mechanism of calculating
cost of equity. It has unrealistic assumptio ns. If the earnings do not
remain stable, this method is not practical.
d)CAPITAL ASSET PRICING MODEL APPROACH (CAPM):
CAPM model describes the risk -return trade -off for
securities. It describes the linear relationship between risk and
return for securit ies. The risks to which a security is exposed are
divided into two groups, diversifiable and non -diversifiable.
The diversifiable risk can be eliminated through a portfolio
consisting of large number of well diversified securities.
The non -diversifiable risk is attributable to factors that affect
all businesses. Such risks are: -
Interest rate changes
Inflation
Political changes etc.
Thus, the cost of equity capital can be calculated under this
approach as:
Ke=R f+b( R m-Rf)
where,
Ke= Cost of equi ty capital
Rf= Rate of return on security
b = Beta coefficient
Rm= Rate of return on market portfolio
Therefore, required rate of return = risk free rate + risk
premiummunotes.in

Page 113

113The idea behind CAPM is that investors need to be
compensated in two ways -time value of money and risk.
The time value of money is represented by the risk -free rate
in the formula and compensates the investors for placing money in
any investment over a period of time.
The other half of the formula represents risk and calculates
the amou nt of compensation the investor needs for taking on
additional risk. This is calculated by taking a risk measure (beta)
which compares the returns of the asset to the market over a period
of time and compares it to the market premium.
Illustration 9:
Calculate the cost of equity capital of Shanthi ltd, whose risk
free rate of return equals 10%. The firm’s beta equals 1.75 and the
return on the market portfolio equals to 15%.
Solution
Ke=R f+b( R m-Rf)
Ke=0 . 1 0+1 . 7 5( 0 . 1 5 –0.10)
=0 . 1 0+1 . 7 5 ( 0 .05)
=0 . 1 8 7 5
=18.75%
5.8 COST OF RETAINED EARNINGS
Like other sources of fund, retained earnings involve cost. It
is the opportunity cost of dividends forgone by shareholders.
The given future depicts how a company can either keep or
reinvest cash or return it to the shareholders as dividends. If the
cash is reinvested, the opportunity cost is the expected rate of
return that shareholders could have obtained by investing in
financial assets.
There are two approaches to measure this opportunity cost.
One approach is by using discounted cash flow (DCF) method and
the second approach is by using capital asset pricing model.
a)by DCF :1
0SDGKP
where,
D1=D i v i d e n d
Po= Current market price
G=G r o w t h ratemunotes.in

Page 114

114b)By CAPM : K S=R f+b( R m–Rf)
where,
Ks= Cost of equity capital
Rf= Rate of return on security
b= Beta coefficient
Rm= Rate of return on market portfolio
Illustration 10: A Ltd provides the following details:
D0=R s4 . 1 9 P0=R s5 0 G= 5%
Calculate the cost of retained earnings based on DCF method.
Solution :

1
0
0
01. 4.19 1.05 0.05.50
0.088 0.05
138
13.8%SDGKP
DGGP
Rs
Rs
 
 



Illustration 11: C Ltd provides the following details:
Rf=7 % b=1 . 2 0 Rm=13%
Calculate the cost of retained earnings based on CAPM
method.
Solution
Ks=R f+b( R m–Rf)
= 7% + 1.20(6%)
=7 %+7 . 2 0
Ks=1 4 . 2 %
5.9 WEIGHTED AVERAGE COST OF CAPITAL (WACC)
WACC (Weighted Average Cost of Capital) represents the
investors’ opportunity cost of taking on the risk of putting money
into a company.
Since every company has a capital structure i.e. what
percentage of funds comes from retained earnings, equity shares,munotes.in

Page 115

115preference shares, debt and bonds, so by taking a weighted
average, it can be seen how much cost/interest the company has to
pay for every rupee it borrows/invest. This is the weighted average
cost of capital.
The weighted average cost of capital for a firm is of use in
two major areas: -
1.In consideration of the firms position;
2.Evaluation of proposed changes necessitating a change in the
firm’s capital. Thus, a weighted average technique may be used
in a quasi -marginal way to evaluate a proposed investment
project, such as the construction of a new building.
Thus, weighted average cost of capital is the weighted
average a fter tax costs of the individual components of firm’s
capital structure. That is, the after tax cost of each debt and equity
is calculated separately and added together to a single overall cost
of capital.
K0= % D(mkt) (K i)( 1–t) + (% Psmkt) K p+ (Csmk t) K e
where,
K0= Overall cost of capital
Ki= Before tax cost of debt
1–t=1 –Corporate tax rate
Kp= Cost of preference capital
Ke= Cost of equity
% Dmkt = % of debt in capital structure
% Psmkt = % of preference share in capital structure
%C s=% of equity share in capital structure
The cost of weighted average method is preferred because
the proportions of various sources of funds in the capital structure
are different. Therefore, cost of capital should take into account the
relative proportions of different sources of finance.
Illustration 12:
Calculate the WACC using the following data
(a) Book value weights Basis
(b) Market value weights Basismunotes.in

Page 116

116The capital structure of the company is as under:
Rs. Debentures (Rs 100 per debenture)
5,00,000
Preference shares (Rs 100 per share)
5,00,000
Equity shares (Rs 10 per share)
10,00,000
The market prices of these securities are:
Debenture Rs 105 per debenture
Preference Rs 110 per preference share
Equity Rs 24 each
Additional information:
1)Rs 100 per debenture redeemable at par, 10% coupon rate, 4%
flotation costs, 10 year maturity.
2)Rs 100 per preference share redeemable at par, 5% coupon
rate, 2% flotation cost and 10 year maturity.
3)Equity shares have Rs 4 flotation cost.
The expect ed dividend is Rs 10 with annual growth of 5%.
The firm has a practice of paying all earnings in the form of
dividend.
Corporate tax rate is 30%.
Solution
Cost of equity =100.0520eK
=0 . 0 5+0 . 0 5
=0 . 1 0
=1 0 %
Cost of Debt =100 9610 1 0.310
100 962dK

7 0.42196
0.0755
7.55% 

Cost of preference shares =25101982pKmunotes.in

Page 117

1175.2
990.05355.35%


a)Calculation of WACC using book value weights
Source of c apital Book value Specific cost (K%) Total cost
10% Debentures 5,00,000 5.55 27,500
5% preference
shares5,00,000 5.3 26,500
Equity shares 10,00,000 10.0 1,00,000
Total 20,00,000 1,54,000
0.1,54,000. 20,00,000RsKRs0.0777.7%
b)Calculation of WACC using market value weights;
5.10 EXERCISES
1.Indicate whether the following statements are true or false :
a)Cost of capital is the cost of borrowing funds.
b)Retained earnings do not have explicit cost.
c)Cost of Preference Share capital is higher than the cost of
equity capital.
d)The higher is the corporate tax rate, the higher is the cost of
debt.
e)Overall cost of capital decreases on payment of entire long
term debt.
(Answers: a -False, b -True, c -False, d -False, e -False)munotes.in

Page 118

1182.What is cost of capital? Explain the problems faced in
determining cost of capital.
3.Explain the different approaches to the calculation of cost of
equity capital.
4.What is weighted average cost of capital ?E x p l a i nt h er a t i o n a l e
behind the use of weighted average cost of capital.
5.Explain the approach to determine the cost of retained earnings.
6.A company has the following specific cost of capital along with
the indicated book and Market Value weigh ts’ :
Calculate the weighted average cost of capital using book
value and market value weights. (Answers : Ko = 9.5%, 9.9% )
7.Two companies, A and B are in the same business and hence
similar operating risks. However, the capital structure of each of
them is different. The following are the details :
Assume that current levels of dividends are generally
expected to continue indefinitely and the income tax rate is 35
percent. Compute the weighted Average Cost of capital of each
company. (Answers : 19%, 17%)
munotes.in

Page 119

119Unit-6
CAPITAL STRUCTURE DECIONS
Unit Structure :
6.0 Objectives
6.1 Introduction
6.1 Meaning of Capital Structure
6.2 Choice of Capital Structure
6.3 Optimum Capital Structure
6.5 Importance of Capital Structure
6.6 Factors Affecting Capita l Structure
6.7 Capital Structure Theories
6.8 Exercises
6.0 OBJECTIVES
After studying the unit the students will know;
The concept of capital structure.
The importance of capital structure.
The concept of optimum capital structure
The choice of capit al structure.
The capital structure theories.
6.1 INTRODUCTION:
Capital structure is the mix of different securities to a firm’s
capitalisation. It is the permanent financing of the company
represented primarily by long -term debt and shareholder’s equity .I t
is also a part of a company’s financial structure. The choice of
capital structure depends upon a number of factors such as nature
of business, regularity of earnings, conditions of the financial
markets and attitudes of the investors. A capital struc ture will be
considered appropriate if it possesses profitability, solvency,
flexibility, conservatism and control. The capital structure of a
company is to be determined initially at the time of incorporation of
a company. The initial capital structure wi ll have long term
implications. It may not be possible to have optimum capital
structure but the management should set a target capital structure
and the initial capital structure should be framed keeping in viewmunotes.in

Page 120

120the target capital structure. Therefore, th e capital structure decision
is a continuous one.
6.2 MEANING OF CAPITAL STRUCTURE:
Capital structure is the mix of a firm’s capitalisation. It
includes long term sources of funds such as debentures, shares,
etc. According to Gavstenberg, capital structu re is the “make -up of
a firm’s capitalisation.” Thus, it represents the mix of different
sources of long term funds, in the capitalisation of the company.
The term capitalisation is used with reference to the total long term
funds raised by a company.
The decisions regarding the form of financing, their
requirements and their relative proportions in the total capital of a
company are known as capital structure decisions. The company
management has to take extreme care and prudence in arriving at
the proper capital structure. The term capital structure is used for
the mix of capitalisation. The capitalisation is used for the sources
of long -term capital of a company. The long term sources of raising
capital are issue of shares, debentures or bonds and long -term
borrowings. The share is a owned capital and debentures and
bonds are borrowed capital. Hence, there should be a mix of
sources of capital.
The capital structure of a company is to be determined
initially, at the time of formation of the company. The initial capital
has long -term implication and hence proper care should be taken
while deciding the sources of capital at the beginning. The capital
structure should be flexible, profitable and simple. The initial capital
structure of a company depends upon many factors.
6.3 CHOICE OF A CAPITAL STRUCTURE:
The choice of an appropriate capital structure depends upon
a number of factors. These factors include nature of company’s
business, regularity of earnings, conditions of financial markets,
attitude of th e management as well as the investors. However, a
firm has the choice to raise funds for financing its projects with the
following choices:
a)Only with equity shares.
b)With equity and preference shares.
c)With equity shares and debentures.
d)With equity shares, preference shares and debentures.
A capital structure will be considered to be appropriate if it
possesses the following features:munotes.in

Page 121

121i)Flexibility :The capital structure should be determined in such a
way that there should be some scope fo r changes according to
the changing circumstances. It should be possible for the
company to provide funds whenever needed for financing its
activities.
ii)Profitability: The capital structure of a company should be
most profitable. The objective of a co mpany is to maximise the
return to the shareholders. Therefore, the capital structure
should tend to minimise cost of financing and at the same time
maximise the returns to the shareholders.
iii)Solvency :The capital structure should be determined in suc ha
way that it should not be a risk of becoming insolvent. Excessive
use of debt or borrowed capital in the capital structure results
into insolvency. It affects profitability as well as liquidity of the
company adversely.
iv)Conservative :The capital structure of a company should be
conservative in the sense ,that the debt portion in the capital
structure should not exceed the limit which the company can
bear. Normally, the debt -equity rat io should not be more than
2:1.
v)Control: While deciding the capital structure of a company, the
management has to see that its control should not be reduced.
The promoter’s control should not be reduced. The promoters
control the company with more proportion of equity than debt. In
order to avoid this, a proper ba lance between owned capital and
debt capital should be maintained.
6.4 OPTIMUM CAPITAL STRUCTURE:
Optimum capital structure is that capital structure at which
the value of equity share is the maximum while the average cost of
capital is the minimum. The value of equity share mainly depends
upon the earnings per share. The theory of capital structure deals
with the issue of the right mix of debt and equity in the long -term
capital of the company. If a company raises debt, the value of
equity shares goes up to a certain point. If the debt increases
beyond that point, the value of equity shares goes down. Therefore,
the company should determine its appropriate level of debt -equity
mix which is known as optimum capital structure.munotes.in

Page 122

1226.5 IMPORTANCE OF CAPITAL STRUCTURE:
The capital structure decisions are very important in financial
management. These decisions influence debt -equity mix which
ultimately affects shareholders.’ return and risk. Since the cost of
debt is cheaper, companies prefer to borrow. The v alue of equity
depends upon earnings per share. As long as return on investment
is more than the cost of borrowings, extra borrowings will increase
the earnings per share. However, beyond the limit, it increases the
risk and the share price may fall. The e ffect of fall in share price
due to heavy load of debt is difficult to measure. Market factors are
so highly psychological and complex as they hardly follow these
theoretical considerations. However, a company can determine an
appropriate debt -equity mix e mpirically, considering various factors.
The debt -equity mix in the capital structure is one of the
important factors. Affecting the value of a share of a company.
There is a significant relationship between the share price and the
variables like return, risk, growth size and leverage. Companies in
India are now showing almost an equal preference for debt and
equity in designing their capital structure. This is due to the freedom
in paying dividend and easy to raise money. However, the returns
have become uncertain due to increasing competition.
An important function of financial management is to decide
an appropriate capital structure of their company. The financial
performance of a company depends upon the capital structure
decisions. A good capital str ucture will help the company to
increase profits, efficiency and reputation of the company.
Therefore, capital structure decisions are very important.
6.6 FACTORS. AFFECTING CAPITAL STRUCTURE:
An appropriate capital structure can be determined on the
basis of the following factors:
1) TRADING ON EQUITY:
Trading on equity means use of owned capital as well as
borrowed capital in the capital structure of a company. A company
can raise funds by issue of shares and debentures. Debentures
carry a fixed rate o f interest and the interest is paid irrespective of
profits. A company can also raise capital only by issue of shares. In
this case, the shareholders will get less amount of dividend
because of large number of shareholders. However, if a company
issues sha res as well as debentures, the shareholders will be
benefited more in the form of dividend. Debenture holders have a
limited share in the company’s profits and hence want to bemunotes.in

Page 123

123protected in terms of earnings and values represented by equity
capital. Fixed interest on debt does not vary with the firms’ earnings
before interest and tax, a magnified effect is produced on earnings
per share.
Illustration 1:
A Ltd wants to raise Rs. 1, 00,000 as capital. The company
expects earnings before interest and taxes (E BIT) Rs. 40,000 per
annum. The management is considering the following alternatives
for raising the capital:
a)Issue 10,000 equity shares of Rs. 10 each.
b)Issue 5000 equity shares of Rs. 10 each and 500, 12%
preference shares of Rs. 100 each.
c)Issue 5000 equity shares of Rs. 10 each and 10% Debentures
of Rs. 50,000.
You are required to calculate earnings per share and advise
the alternative to be used for raising capital, assuming tax rate of
30%.
Solution:
Calculation of earnings per share:
In case of alternative (c) i.e. capital structure consisting of
debt-equity (trading on equity) the earnings per share is highest,
hence the alternative (c) should be followed in order to maximize
the return to shareholders.
2) LEVERAGES:
Leverage is the a bility of a firm to use fixed cost assets or
funds to magnify the return to its owners. There are two leverages
associated with the capital structure i.e. operating leverage and
financial leverage. Operating leverage exists when a firm has amunotes.in

Page 124

124fixed cost tha t must be incurred regardless of volume of business.
On the other hand, financial leverage is a mix of debt and equity in
the capitalisation of the firm. In order to decide proper financial
policy, operating leverage may be taken into consideration as the
financial leverage is a superstructure built on the operating
leverage. The operating profits i.e. earnings before interest and
taxes (EBIT) serves as a function in defining these two leverages.
Financial leverage represents the relationship between the fi rms’
earnings before interest and taxes and earnings available for equity
holders. When there is an increase in EBIT there is a
corresponding increase in market price of equity shares. However,
increased use of debt in the capital structure has certain lim itations.
If debt capital is employed in greater proportion, marginal cost of
debt will also increase and share price may fall as investors may
find it risky. On the other hand, in spite of increased risk, market
price of shares may increase due to specula tion. Therefore, before
using financial leverage, its impact on Earning Per Share (EPS)
must be considered. A company having higher operating leverage
should use low financial leverage and vice versa otherwise, it may
face problems of insolvency and inadeq uate liquidity.
Illustration 2:
GTL Ltd, a widely held company is considering a major
expansion of its production facilities and the following alternatives
are available:
Expected rate of return before tax is 25%. The rate of
dividend of the company i s not less than 20%. The company at
present has low debt. Corporate tax is 30%. Which of the
alternatives you would choose?munotes.in

Page 125

125Solution:
Evaluation of financial alternative (Rs. lakhs)
Alternative (C) is more profitable because shareholde rs will
be benefited more. Therefore alternative (C) should be chosen.
3) INTEREST COVERAGE RATIO:
The ability of a firm to use debt in the capital structure may
be judged in terms of interest coverage ratio. It is the ratio or
relation between operating profit and interest. Higher the ratio,
greater is the certainty of meeting interest payment. If the ratio is
lower, the firm may not be able to pay interest in future.
4) CASH FLOW ANALYSIS:
EBIT -EPS analysis is a good supporting tool in determining
a sui table capital structure. Cash flow under adverse situation
should be examined in order to determine the debt capacity. A high
debt-equity ratio may not be risky if the company has the ability to
generate adequate cash flows. It may be possible to increase the
debt until cash flows equal to the risk set out by debt capital. With
the help of information available, a range can be determined for an
optimum level of debt in the capital structure.
Illustration 3:
BEST Ltd, a profit making company has paid up cap ital of
Rs. 100 lakhs consisting of 10 lakhs equity shares of Rs. 10 each.
Currently it is earning an annual pre -tax profit of Rs. 60 lakhs. The
company’s shares are listed and quoted in the range of Rs. 50 to
Rs. 80. The management wants to diversity prod uction and has
approved a project which will cost Rs. 50 lakhs and it is expected to
yield a pre -tax income of Rs. 40 lakhs per annum. To raise this
additional capital, the following options are under consideration of
the management.munotes.in

Page 126

126a)To issue equity ca pital for the entire additional amount. It is
expected that the new shares (face value Rs. 10) can be sold at
a premium of Rs. 15.
b)To issue 16% non -convertible debenture of Rs. 100 each for the
entire amount.
c)To issue equity capital for Rs. 25 la khs (face value Rs. 10) and
16% non -convertible debenture for the balance amount. In this
case, the company can issue shares at a premium of Rs. 40
each.
You are required to advise the management as to how the
additional capital can be raised keeping in m ind that the
management wants to maximise the earning per share to maintain
its goodwill. The tax rate applicable to the company is 30%.
Solution:
Calculation of EPS under three options (Rs. in lakhs)
Option II, i.e. issue of 16% Debentures is most suitable to
maximise the EPS.munotes.in

Page 127

127Illustration 4:
‘Z’ Ltd is currently EBIT of Rs. 12 lakhs. Its present borrowings are:
(Rs. in Lakhs)
The sales of the company are growing and to support this,
the company proposes to obtain an additional bank borrowing as
Rs. 25 lakhs at 15% p.a. The increase in EBIT is expected to be
20%. Calculate the change in interest coverage ratio after
additional borrowings and after your comments.
Solution:
i)The present EBIT is Rs. 12 lakhs
ii)Interest on present borrowin g Rs.
Term Loans -12% of Rs. 40 lakhs =4.80 lakhs
Public Deposit -12% of Rs. 15 =1.80 lakhs
Bank Loan -15% of Rs. 35 =5.25 lakhs
Total 11.85 lakhs
iii)Present Interest Coverage RatioEBITInterest
12.00
11.851.01Timesiv)Revised EBIT12012100= Rs. 14.40 lakhs
v)Revised amount of interest = Rs.11.85 lakhs ƒ15% of 25 lakhs
= Rs. 11.85 + Rs. 3.75 la khs
= Rs. 15.6 lakhs
vi)Revised Interest Coverage RatioEBITInterest14.4015.600.92Illustration 5:
Mangalore Chemicals Ltd. requires R s. 25 lakhs for a new
plant. This plant is expected to yield earnings before interest andmunotes.in

Page 128

128taxes of Rs. 5 lakhs. While deciding about the financial plan, the
company considers the objective of maximising earnings per share.
It has three alternatives to fina nce the project by raising debt of Rs.
2,50,000 or Rs. 10,00,000 or Rs. 1,50,000 and the balance in each
case, by issuing equity shares. The company’s share is currently
selling at Rs. 150 but it is expected to decline to Rs. 125 in case the
funds are borr owed in excess of Rs. 10,00,000. The funds can be
borrowed at the rate of 10% up to Rs. 2,50,000 at 15% over Rs.
2,50,000 and up to Rs. 1,00,000 and at 18% over Rs. 10,00,000.
The tax rate applicable to the company is 30%. Which form of
financing should th e company chose?
Solution:
Evaluation of alternative proposals (Rs.)
Earning per share in case of alternative II is highest. Hence,
the company should finance the new plant by raising Rs. 10 lakhs
of Debt @ 15% and issue of equity shares. The company can issue
10,000 Equity shares at Rs. 150 each and raise Rs. 15 lakhs
through equity.
6.7 CAPITAL STRUCTURE THEORIES:
A firm has to maintain an optimum capital structure with a
view to maintain financial stability. The optimum capital structure
can be ob tained when the marked value per share is the maximum.
Therefore, the objective of the firm should be to select a financing
or debt equity mix which will maximise the value of the firm,
optimum leverage can be the mix of debt -equity which maximises
the val ue of a company. In order to achieve this goal, the finance
manager has to follow the theories of capital structure of corporate
enterprises. There are four major theories which explain the
relationship between capital structure, cost of capital and value of
the firm. These are as follows:munotes.in

Page 129

1291)Net Income Approach.
2)Net Operating Income Approach.
3)Modigliani -Miller Approach (MM).
4)Traditional Approach.
However, in order to understand this relationship, the
following assumptions are made:
i)The firm employs only two types of capital i.e. debt and equity
capital.
ii)Taxes are not considered.
iii)The firm pays its earnings in full as dividend. There is no
returned earning.
iv)The firm’s total assets are given and there is no change in the
assets.
v)The firm’s total financing remains constant. The firm can change
its capital structure by interchanging the source of finance.
vi)The operating profit is not expected to change.
vii)The business risk remains constant and it is independent of
capital structure and financial risk.
viii)The firm has a perpetual life. It means the business is a going
concern and it has long life.
ix)All the investors has the same subjective probability distribution
of the future expected operating profits for a g iven firm.
6.7.1. NET INCOME APPROACH (NI):
David Durand, of USA, had suggested this approach.
According to him, capital structure decision is relevant to the
valuation of the firm. It means, a change in the capital structure
causes a corresponding change in the overall cost of capital as well
as the total of the firm.
This approach also suggests that a higher debt content in
the capital structure will result in decline in the overall cost of
capital. This will cause increase in the value of the firm and
consequently increase in the value of equity shares of the
company. The Net Income Approach is based on the following
assumptions:
i)The cost of debt is less than cost of equity.
ii)The debt content does not change the risk perception of the
investors .
Thus, the Net Income Approach suggests that an increase in
financial leverage will lead to decline in the weighted average costmunotes.in

Page 130

130of capital and the value of the firm as well as market price of equity
shares will increase. On the other hand, a decrease in the financial
leverage will cause on increase in the weighted average cost of
capital and a consequent decline in the value as well as market
price of equity shares.
The value of the firm on the basis of Net Income Approach
can be ascertained as follows :
V=S+D
where, V = Value of the firm
S = Market value of equity
D = Market value of Debt
The market value of Equity can be ascertained as follows:eNISK
where, S = Market value of Equity
NI = Earnings available to Equity sh areholders.
Ke= Equity capitalisation rate
Under Net Income approach, the value of the firm will be
maximum at a point where weighted average cost of capital is
minimum. Therefore, the theory suggests maximum possible debt
financing for minimizing the c ost of capital. The overall cost capital
is determined as follows:
Overall cost of capitalEBITValue of the firm
Illustration 6:
The EBIT of Kripa Ltd is Rs. 5,00,000. The company has
10% Debentures of Rs. 20,00,000. The equity capitalisation rate is
15%.
You are required to calculate:
i)Market value of Equity
ii)Value of the Company
iii)Overall cost of capital.munotes.in

Page 131

131Solution:
Statement showing value of firm
Equity capitalisation rate = (K e)=1 5 %
i)Market value of EquityeNIK
3,00,00010015 
= Rs. 20,00,000
ii)Value of the company =V a l u eo fE q u i t y+V a l u eo fD e b t
= Rs. 20,00,000 + 20,00,000
= Rs. 40,00,000
iii)Overall cost of capital100EBIT
Value of the firm 
5,00,00010040,00,000 
=1 2 . 5 %
Illustration 7:
Zed Ltd is expecting on annual EBIT of Rs. 10,00,000. The
company has Rs. 40 lakhs in 10% Debentures. The cost of equity
capital or capitalisation rate is 12.5%. You are required to calculate
the tota l value of the company and overall cost of capital.
Solution:
Statement showing value of the firm
Equity capitalisation rate = 12.5%munotes.in

Page 132

132a)Market Value of Equity (s)eNIK6,00,00012.5%
=Rs. 48,00,000
b)Market Value of Debt is Rs. 40,00,000
c)Value of the firms = S+D
= Rs. 48,00,000 + Rs. 40,00,000
= Rs. 88,00,000
d)Overall Cost of capital100EBIT
Value of the firm 
10,00,00010088,00,000 
=11.36%
Illustration 8:
‘H’ Ltd is expecting annual EBIT of Rs. 10,00,000. The
company has issued 10% Debentures of Rs. 40,00,000. The equity
capitalisation rate is 12.5%. The company desires to redeem
debentures of Rs. 10,00,000 by issuing additional equi ty shares of
Rs. 10,00,000. You are required to calculate the value of the firm
and also the overall cost of capital.
Solution:
Statement showing the value of the firm
Equity capitalisation Rate = K e=1 2 . 5 %
a)Market value of Equity (S)eNIK7,00,00012.5%
= Rs. 56,00,000munotes.in

Page 133

133b)Market value of the firm = S + D
= Rs. 56,00,000 + 30,00,000
= Rs. 86,00,000.
c)Overall cost of capital100EBIT
Value of the firm 
10,00,00010086,00,000 
=1 1 . 6 3 %
Illustration 9:
The operating income of ‘A’ Ltd is Rs. 6,00,000. The firms
cost of debt is 10%. The amount of Debt is Rs. 15,00,000. The
overall cost of capital of the firm is 15%. You are required to
determine:
a)Totalv a l u eo ft h ef i r m
b)Cost of equity
Solution:
(a)
Statement sharing the value of the firm:
Earnings before Interest Taxes Rs. 6,00,000
Less: Interest on Debentures
10% of Rs. 15,00,000 1,50,000
Net Income 4,50,000
Total cost of capital 15%Value of the firm =eEBITK=6,00,0000.15= Rs. 40,00,000Market value of Equity =V–D
= Rs. 40,00,000 –15,00,000
= Rs. 25,00,000
b) Calculati on of cost of Equity
1004,50,00010025,00,00018%eNet IncomeCost of Equity KMarket Value of Equity  
 munotes.in

Page 134

1346.7.2 NET OPERATING INCOME APPROACH (NOI):
This approach was also suggested by Mr. David Durand. Net
operating Income means earning before interest and tax. This
approach suggests that the market value of t he firm is not at all
affected by the capital structure changes. The capital structure
decisions of the firm are irrelevant. And change in the leverage will
not lead to any change in the total value of the firm and the market
price of the shares. The marke t value of the firm is ascertained by
capitalising the net operating income at the overall cost of capital
(K) which is considered to be constant. The market value of equity
is ascertained by deducting the market value of the debt from the
market value of the firm.
The net operating income approach is based on the
following assumptions:
1)The overall cost of capital (K) remains constant for all degree of
debt-equity mix.
2)The market capitalises the value of the firm as a whole and
therefore, the spit between debt and equity is not relevant.
3)The low cost debt increases the risk of equity shareholders. This
results in increase in equity capitalisation rate. An increase in
the use of debt is offset by an increase in the equity
capitalisation rate.
The value of the firm is determined as follows:EBITVK
Where, V = Value of the firm
K = Overall cost of Capital
EBIT = Earning before interest and tax.
The value of equity can be determined by using the following
formula:
S=V–D
Where, S = Value of Equity
V=V a l u eo ff i r m
D=V a l u eo fD e b t
Illustration 10:
‘X’ Ltd has an EBIT of Rs. 10 lakhs. The cost of Debt is 10%
and the outstanding debt amounts to Rs. 3,00,000. If the overall
capitalisation rate is 12.5%, calcula te the total value of the firm and
equity capitalisation rate.munotes.in

Page 135

135Solution:
(a)
Statement showing the value of the firm
(Net Operating Income Approach)
Rs.
EBIT 10,00,000
Overall capitalisation rate = 12.5%Market Value of the firmEBITK10,00,00012.5%
Value of Debt 80,00,000Value of Equity 30,00,000
50,00,000
Equity Capitalisation Rate100eEBIT IKVD 
10,00,000 3,00,00010080,00,000 30,00,000
7,00,00010050,00,00014% 
 Illustration 11 :
‘Y’ Ltd has an EBIT of Rs. 30,00,000. Its cost of debt is
12.5% and the outstanding debt is Rs. 40,00,000. The overall
capitalisation rate is 15%. The company decides to raise a sum of
Rs. 10,00,000 through issue of equity shares and use the proceeds
toredeem the debt. You are required to calculate the total value of
the firm and equity capitalisation rate.
Solution:
(a)
Statement showing the value of the firm
munotes.in

Page 136

136(b) Equity Capitalisation Rate100EBIT IVD 
30,00,000 3,75,0001002,00,00,000 30,00,000
26, 25,0001001,70,00,00015.44% 
 6.7.3 MODIGILIANI -MILLER APPROACH (MM):
Modigiliani –Miller approach provides behavioural
justification for constant overall cost of capital and total value of the
firm. It does not provide operational justification for irrelevance of
the capital s tructure in the valuation of the firm. According to this
approach the value of a firm is independent of its capital structure.
MM approach maintains that the average cost of capital does not
change with change in the debt -weighted equity mix or capital
structure of the firm.
The three basic propositions of the MM approach are as
follows:
1)The overall cost of capital (K) and the value of the firm (V) are
independent of the capital structure. In other words ‘K’ and ‘V’
are constant for all levels of debt –equity mix. The total market
value of the firm is given by capitalising the expected net
operating income (NOI) by the rate appropriate for that risk
class.
2)The cost of equity (K e) is equal to capitalisation rate of a pure
equity stream plus a premium for the financial risk. The financial
risk increases with more debt content in the capital structure.
Thus, (K e) increases in a manner to offset exactly the use of a
less expensive source of funds represented by debt.
3)The cut -off rate of investment p urposes is completely
independent of the way in which an investment is financed.
MM approach is based on the following assumptions:
1)Capital markets are perfect. This means investors are rational
and are well informed.
2)All the firms within the same risk class will have the same
degree of business risk.
3)All investors have the same expectation of a firm’s net operating
income with which to evaluate the value of any firm.
According to MM approach the total investment value of a
firm depends upon its underlying profitability and risk. The
operational justification of MM approach can be explained throughmunotes.in

Page 137

137the functioning of the arbitrage process. Arbitrage refers to buying
asset or security at lower price in one market and selling it at a
higher pri ce in another market. As a result equilibrium is attained in
different markets. For example, there are two identical firms. One
has debt in its capital structure and other is not having the debt.
Investor of the firm whose value is higher will sell their s hares and
buy the shares of the firm whose value is lower. They will be able to
earn the same return at lower outlay with the same perceived risk
or lower risk. They would, therefore, be better off. The value of the
leveraged firm can neither be greater no rl o w e rt h a nt h a to fa n
unleaveraged firm. Thus, there is neither advantage nor
disadvantage in using debt in the firm’s capital structure.
Illustration 12:
Two firms A and B are identical in all respect except the firm
A has 10% Debentures of Rs. 5, 00 ,000. Both the firms have the
same earnings before interest and tax accounting to Rs. 1, 00,000.
The equity capitalisation rate of firm A is 16% and that of B is
12.5%. You are required to calculate the total market value of each
of the firms.
Solution:
Statement showing the total value of the firms
Illustration 13:
The two companies X Ltd and Y Ltd are having same
earnings before interest and taxes of Rs. 2,00,000. Y Ltd is levered
company having a debt of Rs. 10,00,000 @ 9% rate of interest. Themunotes.in

Page 138

138cost of equity of X Ltd is 10% and that of Y Ltd is 11.50%. You are
required to calculate the total value of each company.
Solution:
Statement sharing the total value of firms
6.7.4 TRADITIONAL APPROACH:
Traditional approach favors that as a result of fi nancial
leverage up to a certain level cost of capital comes down and value
of the firm increases. However, beyond that level reserve trend
emerges. Thus, the essence of the traditional approach lies in the
fact that a firm through judicious use of debt -equity mix can
increase its total value and thereby reduce its overall cost of capital.
It is because debt is a cheaper source of funds as compared to
raising money through shares because of tax advantage. However,
raising debt beyond a certain point may bec ome a financial risk and
would result in higher equity capitalisation rate.
The principal implication of traditional approach is that the
cost of capital is independent on the capital structure and there is
an optimal capital structure which minimises co st of capital. At the
optimal capital structure the real marginal cost of debt and equity is
the same. Before the optimal point the real marginal cost of debt is
less than real marginal cost of debt is more than the real marginal
cost of equity and beyond this point the real marginal cost of debt is
more than the real marginal cost equity. Therefore, the firm should
strive to reach the optimal capital structure and its total valuation
through a judicious use of the debt and equity capital in the capital
structure. At the optimal capital structure the overall cost of capital
will be minimum and the value of the firm is maximum.munotes.in

Page 139

139Illustration 14:
In considering the most desirable capital structure for a
company the following estimates of the cost of debt and e quity
capital (after tax) has been made at various levels of debt –equity
mix.
You are required to determine the optimal debt equity mix for
the company by calculating composite cost of capital.
Solution:
Composite Cost of Capital
Optimal debt -equit y mix is 30% and 70% i.e. 30%. Debt and
70% Equity, where the composite cost of capital is 10.75% which is
the minimum.munotes.in

Page 140

1406.8 EXERCISES:
1)State whether the following statements are True or False:
a)The optimum capital structure is obtained when th e market
value per equity share is the maximum.
b)The traditional approach is a mid way approach between net
income approach and net operating income approach.
c)The value of a levered firm is higher than that of an
unlevered firm on account of corpo rate taxes.
d)According to MM approach, the value of a firm is affected by
the debt -equity mix.
(Ans.: (a) True, (b) True, (c) True, (d) False)
2)Choose the right answer from the following:
a)When establishing their optimal capital structure fir ms should
strive to:
i)minimise the weighted average cost of capital.
ii)minimise the amount of debt financing.
iii)maximise the marginal cost of capital.
b)A highly leveraged firm is _________ risky than its peers.
i)less ii) more ii i) not
c)An advantage of debt financing is ____________
i)interest payments are tax deductible.
ii)lowers the cost of capital.
iii)does not dilute owner’s earnings.
iv)all the above.
d)An EBIT –EPS indifference analysis chart is use df o r :
i)Evaluating the effects of business risk on EPS.
ii)Examining EPS results for alternative financing plans of
varying EBIT analysis.
iii)Determining the impact of a change in sales on EBIT.
(Ans.: (a) -i, (b)-ii, (c) -iv, (d) -ii)
3)What is capital structure? What is optimum capital structure?
4)Write short Notes on:
a)Weighted Average Cost of Capital.
b)Marginal Cost of Capital
c)M.M.Approach.
d)Traditional Approach.munotes.in

Page 141

1415)AL t dp r o v i d e sy o ut h ef o l l o w i n gf i g u r e s
The company has undistributed reserve of Rs. 3,00,000. The
company needs Rs. 10,00,000 for expansion. This amount will earn
the same rate as funds already employed. You are further informed
that a debt -equity ratio higher than 35% pulls the PE ratio do wn to 8
and raises the interest rate on additional amount borrowed at 14%.
You are required to ascertain the probable price of the share if:
i)The additional funds are raised as loans.
ii)The additional funds are raised by issuing equity shares.
6)E Ltd is considering three financing options. The key information
is as follows:
a)Total Investment is to be raised Rs. 2,00,000.
b)Plans of financing.
c)Cost of Debenture is 8% and cost of preference shares is
10%
d)Tax rate is 35%
e)Equity Shares of face value of Rs. 10 each will be issued at
a premium of Rs. 10 per share.munotes.in

Page 142

142f)Expected EBIT will be Rs. 80,000. You are required to
determine for each plan:
i)Earning per share
ii)The financial break -even point.
iii)Compute the EBIT range among the plans of
indifference.
7)From the following data find out the value of each firm as per
the Modigiliani –Miller Approach:
Each firm expects 12% return on investment.
8)From the following data, determine the value of the firm ‘X’ and
‘Y’ belonging to the homogeneous risk class under (a) the NI
approach and (b) the NOI approach.
Which of the two firms has an optimal capital structure under the
a)NI approach and b) NOI approach.
9)Determine the optimal capital str ucture of Z Ltd from the
following information supplied to you assuming 35% tax rate:
munotes.in

Page 143

14310)The values of two firms X and Y in accordance with the
traditional theory are given below:
Compute the values of the firms X and Y as per the MM
Approach. As sume that corporate income -tax does not exist and
the equilibrium value of Ko is 12.5%.

munotes.in

Page 144

144Unit-7
LEVERAGES
Unit Structure :
7.0 Objectives
7.1 Introduction
7.2 Meaning of Leverage
7.3 Types of Leverages
7.4 Significanc eo fL e v e r a g e s
7.5 Exercises
7.0 OBJECTIVES
After studying the unit the students will:
The meaning of leverage
Business risk & financial risk
Sources of financing
Types of leverages
Importance of leverages
7.1 INTRODUCTION:
A company can raise funds required for investment either by
increasing the owners’ claims or creditors’ claims. The claims of the
owners increase when a company raises funds by issuing equity
shares. The claims of the creditors increase when the funds are
raised by borrowings. Thu s, the various means used to raise the
funds represent the capital structure of the company. The capital
structure decision is of great importance for the management
because it influences the debt -equity mix of the company which
affects the shareholders’ r eturn and risk. If the borrowed funds are
more in the capital structure of a company, it results in an increase
in shareholders’ earnings together with increase in their risk. It is
because the cost of borrowed funds is less than that of the
shareholders’. The costs on account of borrowed funds are
allowable as a deduction for income -tax purpose. However, the
borrowed funds carry a fixed rate of interest which has to be paid
whether the company is earning profit or not. Thus, the risk of the
shareholders in creases in case there are a high proportion of
borrowed funds in the total capital of a company. If the proportion ofmunotes.in

Page 145

145the shareholders’ funds is more than the proportion of the borrowed
capital, the return as well as the risk of the shareholders will be
less. The effect of financing or debt -equity mix on the shareholder’s
earnings and risk can be examined by using the concept of
leverage.
7.2 MEANING OF LEVERAGE:
The term leverage refers to a relationship between two
interrelated variables. It represent s the influence of one financial
variable over some other related financial variable. Leverage is
used to describe the firm’s ability to use fixed cost assets or funds
to magnify the return to its owners.
James Horne defined Leverage as “the employment of an
asset or funds for which the firm pays a fixed cost or fixed return.”
Leverage results when a firm employs an asset or source of funds
which has a fixed cost. There will be no leverage, if a firm is not
required to pay a fixed cost. The fixed cost or return has to be paid
or incurred irrespective of the volume of output or sales, the size of
such cost or return has considerable influence on the amount of
profits available for the shareholders. When the volume of sales
changes leverage helps in quantif ying such influence. Thus,
leverage can be defined as “relative change in profits due to a
change in sales.” A high degree of leverage means large change in
profits due to a relatively small change in sales. Thus, higher the
leverage, higher is the risk an d higher is the expected return.
7.3 TYPES OF LEVERAGE:
There are three commonly used measures of leverage in
financial analysis. These are as follows:
7.3.1 OPERATING LEVERAGE:
The operating leverage is defined as the employment of an
asset with a fixed cost in the hope that sufficient revenue may be
generated to cover all the fixed and variable costs. It can also be
defined as “the tendency of the operating profit to vary
disproportionately with sales.” It exists when the firm has to pay
fixed cost regardless of volume of output or sales. Thus, operating
leverage is a function of three factors:
i)Fixed amount of cost.
ii)Variable contribution margin.
iii)Volume of sales.
The operating leverage can be calculated by using the
following formula:munotes.in

Page 146

146ContributionOperating leverage =Operating profitC=EBIT
Contribution = Sales -Variable Cost.
Operating profit means Earnings before Interest and Taxes (EBIT).
Operating leverage is the ratio of net operating income
before fixed charges to net operating income after fixed char ges.
Degree of Operating Leverage:
The degree of operating leverage may be defined as a
percentage change in the profits resulting from a percentage
change in the sales. It can be put in the form of a formula as
follows:Percentage change in net operating incomeDOL =Percentage change in sales 
Operating leverage is directly proportional to business risk. It
indicates the impact of change in sales on operating income. If a
firm has a high degree of operating leverage a small change in
sales will have a large effect on operating income. The opera ting
profits of such a firm will increase at a faster rate than the increase
in sales. Similarly, the operating profits of such a firm will suffer a
greater loss as compared to reduction in its sales. Generally, the
firms should not operate under condition so fah i g hd e g r e eo f
operating leverage because it is a very risky situation where a small
decline in sales will affect its profits.
Illustration 1:
A company produces and sells 10,000 calculators. The
selling price per calculator is Rs. 500. Variable cost per calculator is
Rs. 200 and fixed operating cost is Rs. 20, 00,000. You are
required to calculate:
a)Operating leverage.
b)If sales are up by 10%, what is its impact on EBIT?
Solution:
a)Statement of Profitability:
munotes.in

Page 147

147ContributionOperating leverage OL =EBIT
30,00,000== 3 T i m e s10,00,000

b)If sales are up by 10%:
%Δ in EBITOL =%Δ insales31030%XX
Thus, if sales are up by 10% the EBIT will increase by 30%
(10x3)which is checked as follows:
3,00,000Increase in EBIT = 10010,00,000  
7.3.2 FINANCIAL LEVERAGE:
The financial leverage can be d efined as “the tendency of
the residual net income to vary disproportionately with operating
profit. It may also be defined as the use of funds with a fixed cost in
order to increase earnings per share of the company.” The financial
leverage indicates the change that takes place in the taxable
income as a result of change in the operating income. It signifies
the existence of fixed interest bearing securities in the capital
structure of the company. Financial leverage induces the use of
funds obtained at a fixed cost in the hope of increasing the return to
he equity shareholders. The financial leverage can be computed
using the following formula:EBITFinancial leverage =EBT
Where, EBIT is the Earnings before Interest and Taxes.
EBT is the Earnings before Tax.munotes.in

Page 148

148Degree of Financial Leverage (DFL) is the ratio of the
percentage change in earning before tax to the percentage
increase in operating profit i.e. EBIT. This can be put in the
following formula:Percentage change in taxable incomeDFL =Percentage change in the operating income 
According to Gitma n, “financial leverage is the ability of a
firm to use fixed financial charges to magnify the effects of changes
in EBIT on the company’s earning per share.” Thus, the financial
leverage indicates the percentage change in earning per share in
relation to a percentage change in EBIT. Accordingly, the degree of
financial leverage can be calculated as per the following formula:Percentage change in EPSDFL =Percentage change EBIT
There will be no financial leverage if the result of the above
equation is less than 1.
Financial leverage is also termed as ‘Trading on Equity’. The
concept of trading on equity states that the company uses equity
capital as well as borrowed capital while deciding its capital
structure. The objective of the term trading on equity is to provide a
higher return to the shareholders of the company. However, trading
on equity should be used for the term financial leverage only when
the financial leverage is favourable. The financial leverage has
potentiality of increasing the return to equity shareholders but at the
same time it cerates additional risk for the shareholders also.
Illustration 2:
Z Ltd. has given the following details:
It has borrowed Rs. 10,00,000 @ 15% p.a. and its equity
share capital is Rs.10,00,000
You are required to calculate :
a)Operating leverage.
b)Financial leverage.munotes.in

Page 149

149Solution:
a)Income Statement:
ContributionOperating leverage OL =EBIT
24,00,000=12,00,000=2 Times


b)EBITFinancial leverage =EBT
12,00,000=10,50,000
1.14Times 
7.3.3 COMBINED LEVERAGE:
Combined leverage expresse s the relationship between
revenue on account of sales and the taxable income. It may be
defined as “the potential use of fixed costs, both operating and
financial which magnifies the effect of sales volume on the earnings
per share of a company.” Thus, de gree of combined leverage is the
ratio of percentage change in earning per share to the percentage
change in sales. It indicates the effect of the change in sales on
earning per share.
Operating leverage and financial leverage are closely
concerned with the firm’s capacity to meet its fixed costs, both
operating and financial. If both the leverages are combined, the
result obtained will disclose the effect of change in sales over
change in taxable profit. Combined leverage can also be called as
composite leverage. It helps to find out the resulting change in
taxable income due to change in sales. The following formula can
be used to find out combined leverage:munotes.in

Page 150

150Combined leverage =O p e r a t i n gL e v e r a g e xFinancial Leverage =Contribution EBIT=EBIT EBTContribution=EBT
The degree of combined leverage can also be calculated as
follows:Percentage change in EPSDCL =Percentage change in sales
Degree of combined leverage indicates the effect of change
in sales on the earning per share.
Illustration 3:
The Income Statement of CRL Ltd. is giv en below: You are
required to calculate
a)Operating leverage,
b)Financial leverage, and
c)Combined leverage.
Income Statement for the year ended 31 -12-2008
Solution:
Income Statement for the year ended 31 -12-2008
munotes.in

Page 151

151
a)Operating LeverageContribution=EBT6,00,000=5,00,0001.2 Timesb)Financial Leverage =EBITEBT5,00,0003,60,0001.39 Timesc)Combined LeverageContribution=OL FL=EBT6,00,0001.2 1.393,60,0001.677.4 SIGNIFICANCE OF LEVERAGE:
Leverages are the tools used by the financial experts to
measure the return to the owners. The financial leverage is
considered to be superior of these tools . Financial leverage focuses
the attention on the market price of the share. The management of
a company always tries to increase the market price of the shares
by increasing the net worth of the company. Therefore, the
management resorts to trading on equ ity in order to increase EBIT
and then the corresponding increase in the price of the equity
shares.
A company has to keep the balance between the two
leverages because they have got tremendous effect on EBIT and
EPS. A right combination between the two leverages is a very big
challenge for the company managements. A proper combination of
both operating and financial leverages is a blessing for the
company’s growth. However, an improper combination may prove
to be a curse. Financial or operating leverage s exist only when the
result of the calculation is more than one.munotes.in

Page 152

152Ah i g hd e g r e eo fo p e r a t i n gl e v e r a g et o g e t h e rw i t hah i g h
degree of financial leverage makes the position of the company
very risky. In this case, a company employs excessively assets for
which it has to pay fixed costs and at the same time it uses a large
amount of debt capital. The fixed costs for using assets and fixed
interest charges bring a greater risk to the company. If the earnings
fail, the company may not be in a position to meet its fixed costs.
Greater fluctuations in earnings are likely to occur on account of the
existence of a high degree of operating leverage. The existence of
high degree of operating leverage will result in a more than
proportionate change in operating profi ts even on account of small
change in sales. The presence of a high degree of financial
leverage causes more than proportionate changes in EPS even on
account of a small change in EBIT. Thus, a company having a high
degree of financial leverage and a high degree of operating
leverage has to face the problems of inadequate liquidity or even
insolvency in one or the other way. However, lower leverages
indicate the cautious policy of the management but the firm may be
losing many profit -earning opportunities. Therefore, a company
should make all possible efforts to combine the operating and
financial leverage in a way that suits the risk -bearing capacity of the
company. Thus, a company with high operating leverage should
have low financial leverage so that the combined leverage may be
ideal. Similarly, a company having a low operating leverage will
stand to gain by having a high financial leverage provided it has
enough profitable opportunities for the employment of borrowed
funds. Low operating leverage and a l ow financial leverage is
considered to be an ideal situation for the maximization of the
profits with minimum of risk.
7.5 SOLVED PROBLEMS
Illustration 4:
‘B’ Ltd. has the following balance sheet and income
statement:
Balance Sheet as on 31 -3-2009
munotes.in

Page 153

153Income statement for the year ended 31 -3-2009
Required:
a)Determine the degree of operating, financial and combined
leverages at the current sales level, if all operating expenses other
than depreciation are variable costs.
b)If total assets remai n at the same level, but sales:
i)increase by 20 per cent and
ii)decrease by 20 per cent.
iii)What will be the earnings per share at the new sales levels?
Solution:
a)
Income Statement
i)Operating leverageContribution=EBIT5,00,0004,40,0001.14 Timesmunotes.in

Page 154

154ii)Financial leverageEBIT=EBT4,40,0003, 40,0001.29 Timesiii)Combined leverage = OL xFL
=1 . 1 4 x1.29
=1 . 4 7
Alternatively (CL)C=EBT5,00,0003, 40,0001.47 Timesb)Earning per share at the new sales level:
Illustration 5:
Calculate the Operating Leverage, Financial Leverage and
Combi ned Leverage from the following data under situation I and II
and Financial Plan A and B:munotes.in

Page 155

155
Solution:
a)Income statement:
munotes.in

Page 156

156Comments: Operating leverage under situation II is higher than
situation I. Financial leverage of plan A is higher than Situa tionII.
Combined leverage of Plan A is also higher in situation I. Hence,
the financial leverage is higher than operating leverage. Financial
plan A is riskier in both the situations.
Illustration 6:
The capital structure of Prakash Industries Ltd. con sists of
an ordinary share capital of Rs. 10 lakhs (Rs. 10 each) and Rs. 10
lakh of 10% Debentures. Sales increased by 20% from 1,00,000
units to 1,20,000 units. The selling price is Rs. 10 per unit, variable
cost amounts to Rs. 6 per unit and fixed expens es amount to Rs.
2,00,000. The income tax rate is 30%. You are required to calculate
the following:
i)The degree of operating leverage.
ii)The degree of financial leverage.
iii)The percentage increase in earning per share at 1,00,000 units
and 1,20, 000 units.
You are also required to comment on the behaviour of
operating and financial leverages in relation to increase in
production from 1,00,000 units to 1,20,000 units.
Solution:
Income Statement
% increase in EPS = 1.26 –0.70 = 0.56
0.53100 80%0.70munotes.in

Page 157

1574,00,000 4,80,000Operating leverage =2,00,000 2,80,0002 1.712,00,000 2,80,000Financial leverage =1,00,000 1,80,0002Times TimesTimes 

 1.56TimesComments: On account of increase in sales from 1 lakh units to
1,20,000 units, the EPS has increased by 80%. While the operating
leverage h as come down from 2 times to 1.71 times and financial
leverage has also declined from 2 times to 1.56 times. There is a
significant decrease in both the business risk and the financial risk
of the company on account of reduction in both the leverages.
Illustration 7:
A firm has sales of Rs. 75 lakhs, variable cost of Rs. 42 lakhs and
Fixed cost of Rs. 6 lakhs. It has a debt of Rs. 45 lakhs @ 9% and
equity of Rs. 55 lakhs.
i)What is the firms’s ROI?
ii)Does it have favourable financial leverage?
iii)If the firm belongs to an industry whose asset turnover is 3,
does it have a high or low asset leverage?
iv)What are the operating, financial and combined leverages of
the firm?
v)If the sales drop of Rs. 50 lakhs, what will be the EBIT?
vi)At what level of EBT of the firm will be equal to zero?
Solution:
Income Statement
1)Return of Investment (ROI)EBIT= 100Capital Employed
27,00,0001001,00,00,00027% munotes.in

Page 158

1582)The return on investment at 27 % is highe r than the interest
payable on debt at 9%. Thus, the firm has a favourable financial
leverage.
3)Assets TurnoverNet Sales=Total Sales75,00,0001,00,00,0000.75The industry average is 3. Hence, the firm has a low ass et
average.
4)(i)Operating LeverageContribution=EBIT33,00,00027,00,0001.22 Timesii)Financial LeverageEBIT=EBT27,00,00022,95,0001.1764 Timesiii)Combined LeverageContribution=EBT33,00,00022,95,0001.438 Times5)If the sales drop to Rs. 50 lakhs, from Rs. 75 lakhs, the fall is by
33.33%. Hence, the EBIT will drop by 40.66 % (33.33 ₹1.22).
Hence, the new EBIT will be Rs. 27,00,000100 - 40.66%100   Rs. 16,02,180.
6)EBT to become zero means 100% reduction in EBT. The
combined leverage is 1.438. Hence, sales have to drop by
100/1.438 i.e. 69.54%. The new sales will be Rs. 75,00,000
100 - 69.54%100   Rs. 22,84,500.munotes.in

Page 159

159Illustration 8:
Prepare income statements from the data given below for P,
Q and R companies:
Compute net profit (after tax) rate for all the three
companies. Offer your comments on the leverages and pro fitability
position of all the three companies.
Solution:
Income Statement
Comments:
1)Leverage
Combined leverage: 5x4=2 0 4x5=2 0 7x6=4 2
Very high V eryhigh Very very high
2)Profitability: Good Satisfactory Poor
3)Working: Calculation of sales for ‘P’munotes.in

Page 160

160i)DFLEBIT 4=EBT 1Interest is Rs. 45,000EBIT –EBT = Rs. 45,0004E B T=E B I T4E B T –EBT = 45,0003E B T=4 5 , 0 0 0EBT45,000= .15,0003RsEBIT = 15,000 x4 = Rs. 60,000
ii)DOLContribution=EBIT5 Contribution=1 60,000Contribution 5 60,0003,00,000
 

iii)Variable cost as a percentage of sales = 50%
Contribution is Rs. 3,00,000Variable cost is also Rs. 3,00,000Sales = Rs. 6,00,000
Illustration 9:
From the following information available for four companies,
calculate:
i)EBIT
ii)EPS
iii)Operating leverage
iv)Financial leverage
(ICU A/Inter Dec.1996)munotes.in

Page 161

161Solution:
Income Statement
Illustration 10:
The Balance sheet of International T rade Ltd. as on 31st
March, 2008 is as under:
munotes.in

Page 162

162The income assets turnover ratio of the company is 3, its
fixed operating cost is 1/6 of sales and variable operating cost is
50% of sales. The corporate tax rate is 35%.
You are required to calculate:
a)The operating, financial and combined leverages.
b)The market price of the share if the P/E multiple is 2.5.
c)The level of EBIT if the EPS is (a) Rs. 15 and (b) Rs. 25.
Solution:
Workings:
1)Total assets turnover is 3.Total Assets TurnoverNet Sales=Total assets
3Net Sales=300lakhsNet Sales =3x3 0 0
= Rs. 900 lakhs.Fixed operating cost =19006lakhs
= Rs. 150 lakhsVariable operating cost = 50 % of Net sales
=5 0%o f9 0 0 lakhs
= Rs. 450 lakhs
2)
Income Statement
munotes.in

Page 163

163a)i)Operating leverageContribution=EBIT450=3001.5Timesii)Financial leverageEBIT=EBT300=2881.04Timesiii)Combined leverageContribution=EBT450=2881.56 1.5 1.04Times 
b)Calculation of Market Price of the share
P/E RatioMarket Price=EPSMarket price= P/E Ratio xEPS
=2 . 5 x20.78
= Rs. 51.95
c)Calculation of the level of EBIT if the EPS is Rs. 15 and Rs. 25:
Income Statement for EPS:
EPS Rs. 15 Rs. 25No. of shares 9 lakhs 9 lakhsPAT 135 lakhs 225 lakhs
Tax @ 35% 72.69 121.15PBT 207.69 lakhs 346.15 lakhs
Interest 12.00 12.00EBIT 216.69 358.15munotes.in

Page 164

1647.7 EXERCISES:
1)Choose the right answer with your reasoning:
a)The __________ is the percentage change in operating income
that results from a percentage change in sales:
i)Degree of operating leverage
ii)Degree of financial leverage
iii)Degree of combined leverage
b)A highly leveraged firm is __________ risky than its peers.
i)Less
ii)More
iii)Same
c)An advantage of Debt financing is ____________.
i)Lowers the cost of capital
ii)Increa ses the cost of capital
iii)Dilutes owners earnings
d)Combined leverage is the percentage change in relationship
between sales and ___________.
i) Operating income
ii) Operating leverage
iii) Earning per share
e)If interest expenses for a firm rise, we know that the firm has
taken on more __________.
i)Financial leverage
ii)Operating leverage
iii)Combined leverage
2)Define operating leverage and financial leverage. How these
leverages are measured?
3)What is combined leverage? Exp lain its significance in financial
planning of a firm.
4)A firm has sales of Rs. 75 lakhs variable cost of Rs. 42 lakhs
and fixed cost of Rs. 6 lakhs. It has a debt of Rs. 45 lakhs at 9%
and equity of Rs. 55 lakhs.
a)What is its ROI?
b)Does it hav e favourable financial leverage?
c)What are the operating, financial and combined leverages of
the firm?
d)If the sales drop to Rs. 50,00,000, what will be the new
EBIT?munotes.in

Page 165

1655)The Balance Sheet of a company is as under:
Balance sheet as on 31.12.2008
The company’s total assets turnover is 3.00, its fixed operating
costs are Rs. 10,00,000 and variable operating costs ratio is 40%.
The income tax rate is 30%.
Calculate:
a)Operating, financial and combined leverages.
b)Determine the likely le vel of EBIT if EPS is (a) Rs. 10 (b) Rs. 30
and (c) Rs. zero.
6)Calculate the degree of operating leverage degree of financial
leverage and degree of combined leverage for the following
companies and interpret the results.
(7) Find out the financia ll e v e r a g ef r o mt h ef o l l o w i n g :
munotes.in

Page 166

1668)A and B are two companies competing with each other. Their
revenue statements are given below:
With the help of leverages, comment on the business risks of
the two companies.
9)The following particulars r elate to X and Co. Ltd.
Using the concept of combined leverage, by what
percentage will earnings per share increase, if sales increase by
10%? Verify your answer by calculating earnings per share.munotes.in

Page 167

16710)From the following data prepare income statem ent of A, B and
C companies.

munotes.in